Tải bản đầy đủ (.docx) (60 trang)

TUYỂN TẬP CÁC ĐỀ THI ĐẠI HỌC, CAO ĐẲNG TỪ NĂM 2007 ĐẾN 2011 potx

Bạn đang xem bản rút gọn của tài liệu. Xem và tải ngay bản đầy đủ của tài liệu tại đây (757.45 KB, 60 trang )

TUYỂN TẬP CÁC ĐỀ THI ĐẠI HỌC, CAO ĐẲNG
TỪ NĂM 2007 ĐẾN 2011
A. HÓA HỌC ĐẠI CƯƠNG
1. Nguyên tử, bảng tuần hoàn các nguyên tố hóa học – liên kết hóa học: 2
- Tp nguyên tử
Câu 1:  !"#$!%

&#'(# )#"# *#'+# ,#'# !/$*-001
Câu 2: /23!45 !6!4! !67!89!
40#)!:6;!234<66=>!?3#*@=4
&#A )#&B *#BC ,#D!C !/$-E*- !)00+
Câu 3: BFGH+#$6I!%6
4!.6I!%6(#*F4JK258!:!%
LBM''N&M'"NOM'N*M'+NCMP7
&#C*# )#B*# *#&*# ,#&O# !/$*-00(
- Đồng vị
Câu 4: /Q!IR!IRS 
P"
1
*

P
1
*
#B !8TIRP"U#
/JJV6IRS
P
1
*

&#+W )#0 *#UW ,#+"W# !/$*-00+


Câu 5: *F !6!4X3TJK2L'




P
"
'
N'




P
"

N'




P
"

"
'
#,Y
R6F !6!>:!.V?JZ [F!\!L
A. X4# B. 4X# C. X4# D. 4X#5/$*-0'07
Câu 6: B]ISIGI^ !!."L

P  P
'" P '
 4 X _
&#4`6F; )#X` !
*#4` ,#XIRS`6F;5-E&0'07
Câu 7:/Q!!IRS8.L
"+
'+
*
!:6U"Wa!
"
'+
*
#/JW
!K2
"+
'+
*
E*A
U
L
&#(1W )#(U"W *#(PW ,#(+1W5-E !)0''7
- Cấu hình, vị trí, biến thiên tính chất
Câu 8:
,YR6F!
b
4
c

XI.3T'





P

L
&#B
b
*
c
&#)#D!
b
C
c
B# *#B
b
C
c
B# ,#d
b
*
c
&# !/$-E*- !&00+
Câu 9: &!
c

!4
b


I.3TH!`"

"
P
#eSZF
8\JF;
&#=Q'+ fU6egg&5G6Z6egg7N4=Q0 fU6gg&5G
6Z6gg7#
)#=Q'( f"6eg&5G6Z6eg7N4=Q0 fU6gg&5G
6Z6gg7#
*#=Q'+ f"6egg&5G6Z6egg7N4=Q0 fU6gg&5G
6Z6gg7#
,#=Q'( f"6egg&5G6Z6egg7N4=Q0 f"6gg&5G
6Z6gg7# !/$-E*- !&00+
Câu 10: *3T!
b

'




P
"

"
P
"?
P
#/8\JFF;


&# TU6eggg&# )# TU6gg&#
*# T"6eg)# ,# TU6eggg)# !/$-E*- !&001
Câu 11: B      h 6= V K2  3  "# B  4i
h6=VK2"6hH!`#B4 j6#
B4JK2
&# !6! !6!# )#! !6 !6!#
*# !6! Z!:6# ,# Z!:6 !6!# !/$*-001
Câu 12: /66&5G6Z7[6eggg&5G6Z6eggg7!.VI!%Z
Trang 1/59 – Các dạng bài tập Hóa học ôn thi ĐH, CĐ từ 2007 đến 2011
GT
&#IG6I!%!\6?JZ! !6V?J# )#Z! !6!\6?J8F ZV?J#
*#Z !6!V?JIG6I!%V?J#
,#Z !6!V?J8F Z!\6?J# !/$-E*- !)00+
Câu 13: * F   5X M ''7 5X M'+7 4 5XM 17  k 5X M '17# - G6I!%  FV?J
=Q
&#4lllk# )#klll4# *#ll4lk# ,#llkl4#
 !/$*-00+
Câu 14: )F ZFL
"
D!
(
A
1
C
''
BIK2>:=QV?J[F!\!
&#CBAD!# )#CD!AB# *#CAD!B# ,#D!BAC# !/$-E*- !&00(
Câu 15: *FLd5XM'17B5XM+7$!5XM'U75XM'7#,YR6FIK2m>:
!.!\6?J8F Z[F!\!L

&#B$!d# )#d$!B# *#dB$!# ,#d$!B# !/$-E*- !)001
Câu 16: ,YFm>:!.V?JZ! !6[F!\!L
&#OBAC# )#OBCA# *#BOAC# ,#BOCA# !/$-E*- !)00(
Câu 17: D! :F;!nFGE

A! :
A. FS @GQ# B. FSGQ#
C. !# D. !I#5/$*-0'07
Câu 18:*F[D!I:C!.VI!%ZGT
&#)F ZIG6I!%I.V
)#)F ZVIG6I!%!\6
*#)F Z!\6IG6I!%V
,#)F ZIG6I!%I.!\65-E&0'07
Câu 19: !
"b
+1I6I!%!. @6
I!%'1#*3T
&#o&p"?

U
'
# )#o&p"?
P
U

# *#o&p"?
P
U
'
# ,#o&p"?

"
U

#5-E)0'07
- Hóa trị cao nhất với oxi, trong hợp chất khí với H
Câu 20: *@=G23 Z8h!k!IkE
"
#/>!6kFS3T
>!!:6+U0+W. !K2#Bk
&#&# )#B# *#$# ,#O# !/$-E*- !)00(
Câu 21: B3TH!`


U
#/23 Z
H!!I!:61U'W !K2#OJV6 !K2>!3
&#000W# )#++W# *#P000W# ,#U000W# !/$-E*- !&001
c,QIF! :>I! :IGQ! :6!q
Câu 22: E23G! :!
&#BE
U
*# )#E*# *#E

A# ,#BE
"
# !/$-E*- !&00(
Câu 23: ,YR6F3G<! :FSGQL
&#A

E


ABE
"
# )#E

AECE

$# *#E*A
"
E

$# ,#EC*

E

A# !/$*-001
Câu 24: B3T'




P
"

"
P
U
'
43T
'






#D! :F;!n4!! :
&# !6!#)#FS#*#!#,#]# !/$*-00(
Câu 25: OF8!qIGI^_
&#Om3^!q#)#rqmB*R!?KH!?!qG#
*#BKHIF!!qG#,#d!6K3^!qG# !/$-E*- !)001
Câu 26:/8\JFF;h6gg&4h6e&#*@=
23?L
&# 
"
4

)#

4
"
*#

4

,#

4

5*-0''7
- Dự đoán liên kết, xđ số liên kết, độ phân cực liên kết, mạng tinh thể

Câu 27: =IGQ! :F;FGIK2m>:=Q!\6?J[F!\!L
&#E)EgE* )#EgE)E* *#E*E)Eg ,#EgE*E)5*-0''7
Câu 28:*F36GkhôngGQL
&#E)*A

*E
U
# )#*

*A

*

E

#
*#BE
"
)

*

E
U
# ,#E**

E

)


#5-E)0'07
Câu 29:d!FBE
"
H!BE
U
b
F8!q @I^L
&#/BE
"
BE
U
b
!I.>!c"#
)#BE
"
Z8sBE
U
b
Z>!#
*#/BE
"
BE
U
b
!I.S"#
,#OGBE
"
!BE
U
b

I.=t S#5-E !&0''7
Câu 30: OF8!qIGsai_
&#/!qKHIF!q!I.!!qG#
)#/!qB*>u6v!!I.P!K2?3J3#
*#/3\F!qGI. \ 8!#
Trang 2/59 – Các dạng bài tập Hóa học ôn thi ĐH, CĐ từ 2007 đến 2011
,#/!qF! :H!89! :S#5-E !)0''7
2. Phản ứng oxi hóa – khử, tốc độ phản ứng và cân bằng hóa học: 2 + 0,5
- Vai trò oxh – khử, cân bằng PT
Câu 31: *F\=L
7CAbEBA
"

5Iw

7

x 87C$bE

$A
U

5Iw

7

x
7&

A

"

bEBA
"

5Iw

7

x ?7*b??SC*
"

x
7*E
"
*EAbE



y7sb&BA
"

??SBE
"

x
7*

E
U


b)


x 7!>5!>!7b*5AE7


x
,YR6F\=I.!\=>!c L
&#8?y# )#8?y# *#8?# ,#8?# !/$-E*- !&00+
Câu 32: *[3LCCAC5AE7

C5AE7
"
C
"
A
U
C

A
"
C5BA
"
7

C5BA
"
7
"

C$A
U
C

5$A
U
7
"
C*A
"

JK2
\=H!EBA
"

Iw#$\=!\=>!Fc 
&#(# )## *#+# ,#P# !/$-E*- !&00+
Câu 33: *F\=L
*5AE7

b*


→
*A*


E$
b
$A


→
"$bE

A
BA

bBAE
→
BBA"
b
BBA
bE

A Ud*A
"
b
$A

→
"$bE

A
A
"

xA


bA

$\=>!F 
&## )#U# *## ,#"# !/$-E*- !)00(
Câu 34: / %  5F   ! !\7  3 \ F 3  K T \ = !n*H!??S
EBA
"

Iw
&#'0# )#''# *#(# ,#1# !/$-E*- !&00+
Câu 35:
*KTF;LC"AUbEBA"
→
C5BA"7"bB>AbEA
$ !G89K
TF;H!%F3n!!\T%EBA
"


&#'">c1# )#UP>c'(# *#U>c'(# ,#">c1# !/$-E*- !&001
Câu 36: /\=IF*C$


\z6*AC

A
"

$A


T6G*C$



{
&#]'# )#]'"#
*#K|'# ,#K|'"# !/$-E*- !)00+
Câu 37: *8!:F\=>\L
C)


b)

xC)" B)b*


xB*b)
OF8!qI^L
&#/Z )
c

6C
b
# )#/Z>!*


6C
"b
#
*#/Z *
c


6)
c
# ,#/Z>!)


6*

#
 !/$-E*- !)00(
Câu 38: *?YF3!LX$CA$A

B

E**
b
*
c
#$3!\Z>!Z 
&#+# )## *#U# ,#P# !/$-E*- !&001
Câu 39: *?YF3!L*

C

$A

B
b
*
b
C

b
&
"b

b
$
c
*
c
#$3!?YI.
Z>!FZ 
&## )#U# *#"# ,#P# !/$-E*- !)00(
Câu 40: *\=L
B

$A
"

bdA
U

bBE$A
U

xB

$A
U

b$A

U

bd

$A
U

bE

A#
/%F35n!!\7KT\=
A. U+# B. +# C. "'# D. "#5/$*-0'07
Câu 41: B$I!a[3 [3>!F\=IG_
A. $bB
0

→
B

$# B. $bPEBA
"

5Iw7

0

→
E

$A

U
bPBA

bE

A#
C. $b"C


0

→
$C
P
# D. U$bPBAE
5Iw7

0

→
B

$bB

$

A
"
b"E


A#5/$*-0'07
Câu 42:*\=LPC$A
U
bd

*

A
+
b+E

$A
U
→"C

5$A
U
7
"
b*

5$A
U
7
"
bd

$A
U
b+E


A
/\=3>!3 JK2
&#C$A
U
d

*

A
+
#)#d

*

A
+
C$A
U
#*#E

$A
U
C$A
U
# ,#d

*

A

+
E

$A
U#
(CĐ 2011)
Câu 43:/\=Ld

*

A
+
bE*→**
"
b*

bd*bE

A
$GE*I!a3 89 JGE*6!\=#}!FS 
&#Ut+# )#'t+# *#"t'U# ,#"t+#5-E&0'07
Trang 3/59 – Các dạng bài tập Hóa học ôn thi ĐH, CĐ từ 2007 đến 2011
Câu 44:*>6C??S=6E

$A
U
5<%>LML7IK26\z6 
?3??S<=6!y#$6?K2CK| !8S
&#"> )# *#> ,#5-E&0'07
Câu 45: *F\=L

57$bE*5Y7 587C$bE

$A
U
5Y7
57A

bE*5Iw7 5?7*bE

$A
U
5Iw7
57&bE

$A
U
5Y7 57C$A
U
bdA
U
bE

$A
U

$\=6E
b
>!I!a>!L
&#" )#P *# ,#5-E !)0''7
- Tốc độ phản ứng

Câu 46: *3>^FA


'006??SE

A

P0!GIK2""P6 ZA


5hI 7#/I
8T\=5ZE

A

7P0!G
  &# 


0#'0

U
 
6t
5#7#
)#


0#'0




6t5#7#
*#


0#'0

"

6t5#7#
,#


#'0

U

6t5#7#
 !/$-E*- !)001
Câu 47: *KT;\=26!


  "
>
B 5 7b"E 5 7 BE 5 7
→
¬ 
d!VRI!IJI\=]L
&#V(J# )#VJ# *#VPJ# ,#!\6I!J# !/$*-00+

Câu 48: *\=L)

bE*AAExE)b*A

#BRI8IJ)

6tZ0!GRI)

a
!00'6tZ#/I8T\=Z)

U#'0
c
6t5l.s7#}!FS
A. 00'(# B. 00'P# C. 00'U# D. 00'#5/$*-0'07
- Hằng số cân bằng, Chuyển dịch CB
Câu 49: 8T\=?Z @I!=v2 ZB


E


H!RIK=0"0+#
$ !\=2BE
"

IF!G89h

*E



!:60W
qZv2IK2#E9G
89d
*

h

*\=!FS
&#00# )#"'# *#0P01# ,#000# !/$-E*- !&001
Câu 50:
*FG89L
   
   
' '
5'7E 5 7bg 5 7 Eg5 757 E 5 7b g 5 7 Eg5 7
 
' '
5"7Eg5 7  E 5 7b g 5 75U7Eg5 7 E 5 7bg 5
 
→ →
¬  ¬ 
→ →
¬  ¬ 
 
7
57E 5 7bg 57 Eg5 7
→
¬ 
r!%I>FIS:d

*

G895'789PUTd
*

890'G89
&#5U7# )#57# *#5"7# ,#57# !/$*-001
*G89;L$A

5 7bA

5 7
ˆ ˆ†
‡ ˆˆ
$A
"
5 7N\=]\=~!%#OF8!qI^L
&#*G89q?S!.] !V!%I#
)#*G89q?S!.S !!\6RIA

#
*#*G89q?S!.] !!\6F3%\=#
,#*G89q?S!.S !!\6RI$A
"
# !/$-E*- !&00(
Câu 51: *G8958T Z7L
*A5 7bE

A5 7
→

¬ 
*A

5 7bE

5 7•El0
/F:L5'7V!%IN5766K2!KHN5"766K2E

N5U7VF3%N
57?`3>^F#
,YR6F:I.6I!G89%L
&#5'75U757# )#5'7575"7# *#575"75U7# ,#5'7575U7# !/$*-001
Câu 52: *G89F;LB


5 7b"E


5 7
→
¬ 
BE
"

5 7N\=]\=\!%#*G89F
; @8Sq?S !
&#I!RIB

# )#63>^FC#
*#I!!%I# ,#I!F3%# !/$-E*- !)00(

Trang 4/59 – Các dạng bài tập Hóa học ôn thi ĐH, CĐ từ 2007 đến 2011

0

0
Câu 53: *FG89F;L
B


5 7b"E


5 7
→
¬ 
BE
"

5 75'7E


5 7bg


5 7
→
¬ 
Eg5 757
$A



5 7bA


5 7
→
¬ 
$A
"

5 75"7 BA


5 7
→
¬ 
B

A
U

5 75U7
d!I!F3nG89;8Sq?SL
&#5'7575"7# )#575"75U7# *#5'75"75U7# ,#5'7575U7#
 !/$*-00(
Câu 54: *FG89L
5'7
$A
5 7bA


5 7
→
¬ 
$A
"
5 7 57B

5 7b"E

5 7
→
¬ 

BE"
5 7
5"7*A

5 7bE

5 7
→
¬ 
*A5 7bE

A5 7 5U7Eg5 7
→
¬ 
E

5 7bg


5 7
d!I!F36R6FG89F;I. @8Sq?S
&#5'757# )#5'75"7# *#5"75U7# ,#575U7# !/$*-001
Câu 55: *G898T ZLBA


→
¬ 
B

A
U
56GI~7 5 @67
)!: !!%I8TT6GI~?J#O\=]L
&#•El0\=\!% )#•E€0\=\!%
*#•El0\=!% ,#•E€0\=!% !/$-E*- !
&001
E9G89\=>FIS<•
&#!%I# )#F3# *#3>^F# ,#RI# !/$*-00(
Câu 56: *G89F;LO*


5 7O*
"

5 7b*


5 7N

*G89q?S!.] !
A. VF3%\=# B. 6*


%\=#
C. 6O*
"

%\=# D. V!%I%\=#5/$*-0'07
Câu 57: *G89F;LB

(k)b"E

(k)

⇄ BE
"
(k)
E 0
∆ <
*G89q?S!.] !L
&#VF3%\= )#V!%I%\=
*#!\6F3%\= ,#63>^F%\=5*-0''7
Câu 58:*\=LE

(k)bg

(k)⇄ Eg(k)
r!%IU"0
0

*9G89d
*
\=89"1P#-68T Z?Z @I!'0Z
=U06E

U0PU6g

#d!%\=IF!G89hU"0
0
*RIEg
&#0+# )#0"0# *#0# ,#0''# (CĐ 2011)
Câu 59: *G89$A

5 7bA

5 7

$A
"
5 7#d!V!%IT< !v2 ZH!E

!\6I!#OF
8!qI^ !!.G89L
&#O\=S\!%G89?Sq!.] !V!%I#
)#O\=]\!%G89?Sq!.S !V!%I#
*#O\=S!%G89?Sq!.] !V!%I#
,#O\=]!%G89?Sq!.S !V!%I#5-E&0'07
Câu 60:jG89LB

A

U
5 7

BA

5 7h
0
*#d!q?S6F!G896H!:RI
B

A
U
V1JTRIBA

&#V1J# )#V"J# *#VUJ# ,#!\6"J#5-E&0'07
Câu 61: *FG89
5g7Eg5 7 E

5 7bg

5 7N
5gg7**A
"
57 *A57b*A

5 7N
5ggg7CA57b*A5 7 C57b*A

5 7N
5ge7$A


5 7bA

5 7 $A
"
5 7
d!!\6F3%G898Sq?S!.S
&#U )#" *# ,#'5-E)0'07
Câu 62:*G89;LE

5 7bg

5 7

Eg5 7N∆E€0#*G89 @8Sq?S !
&#!\6F3%# )#!\6RIEg#
Trang 5/59 – Các dạng bài tập Hóa học ôn thi ĐH, CĐ từ 2007 đến 2011
*#V!%I%# ,#VRIE

#5-E !&0''7
Câu 63:*G89;L$A

5 7bA

5 7$A
"
5 7N∆El0
*F8!%FL5'7V!%I57VF3%\=5"7!%I5U7?`63>^
Fe


A

57!\6RI$A
"
5P7!\6F3%\=#Bn8!%F6G89q
?S!.]_
&#575"75U75P7 )#5'7575U7 *#5'7575U757 ,#575"7575-E !)0''7
Câu 64: *P6*AU6E

A68T Z?Z @I!'0Z#B8T6|!!h
("0
0
*Iq%II:F!G89L*A(k)bE

A(k)

*A

(k)bE

(k)59G89d

M'7#BRIG89
*AE

AJK2
&#00'(000( )#00'00U
*#00(0'( ,#000(00'((ĐH khối B 2011)
3. Sự điện li: 2 + 0,5
- pH, α, K

a
, K
b
Câu 65: ,?SE*??S*E
"
*AAE`RI6tE!??SK=>#‚%
!n>5!\!:='00G*E
"
*AAET'GI!%!7
&#M'00># )#M># *#M>c# ,#M>b# !/$-E*- !&00+
Câu 66:
*??S=v2R6*E
"
*AAE0'*E
"
*AAB0'#)!:h

*

d


*E
"
*AAE
'+#'0
c

8~uQG!KH#}!FSE??Sh


*
&#'00#)#UU#*#((#,#U+P# !/$-E*- !)001
Câu 67:,?SR6*E
"
*AAE'5d

M'+#'0
c
7E*000'#}!FSE??SL
&#U" )#"" *#'++
Câu 68:*'(6v28R6*&5<%6K=UL'7"06??SR6E

$A
U
0
EBA
"
 !F\=>\IK26 ZBA5\z6 ?3B
b
7#/6
BAH!0'6A

IK2v2 Z4#*84F?•H!E

AIK2'06??SEMs#
}!FSsL
&#' )#" *# ,#U5-E !)0''7
- Vai trò, môi trường dung dịch muối, tồn tại các ion
Câu 69: ,YR6F!`R!6??SL
A. &

"b
OA
U

*
ƒ
)
b
# B. B
b
d
b
AE
ƒ
E*A
"
ƒ
#
C. d
b
)
b
AE
ƒ
*
ƒ
# D. *
b
*
ƒ

B
b
*A
"
ƒ
#5/$*-0'07
Câu 70: ,?SIGE€+_
A. ,?S&

5$A
U
7
"
# B. ,?S*E
"
*AAB#
C. ,?SB*# D. ,?SBE
U
*#5/$*-0'07
Câu 71: *Z??SdAEEM'0(00Z??SE*EM"0IK2??S4EM
''0#}!FSL
&#0' )#'P0 *#'+( ,#0(05*-0''7
Câu 72L,?S=L00+6B
b
N006

U
$A

>6AE

c
#,?S4=
U "
*A  BA
− −
6
E
b
N6
U
*A


"
BA

00U#/4IK2'006??SX#,?SXE58~uQI!%!
E

A7
&#' )# *#' ,#'"5-E&0'07
Câu 73: *  ?Y  F  3L  d&5$A
U
7

#'E

A  *

E


AE  *
'
E

A
''
  
5s7  *E
"
*AAE*5AE7


*E
"
*AABE
U
#$3I!%!
&## )#U# *## ,#"# !/$-E*- !)00(
Câu 74: *?YF3L*5E*A
"
7

BE
U
*5BE
U
7

*A

"
X$A
U
&5AE7
"
X5AE7

#$3?YZ3
K„Z
&#"# )## *## ,#U# !/$-E*- !&00+
Câu 75: * ?Y F 3L *5AE7
"
 &

5$A
U
7
"
 5AE7

 X5AE7

 A *A
"
# $ 3  ?YZ3K„
Z
&## )## *#"# ,#U# !/$*-00(
Câu 76: *F23?Y3?KH!IGI.ZK„Z_
&#*5AE7
"

X5AE7

O85AE7

… )#*5AE7
"
X5AE7

5AE7


*#*5AE7
"
O85AE7

5AE7

 ,#*5AE7
"
C5AE7

5AE7

 !/$*-00+
Câu 77: * F 3L & &

A
"
 &


5$A
U
7
"
 X5AE7

 BE$ d

$A
"
 5BE
U
7

*A
"
# $ 3 I. \ =IK2H!
??SE*??SBAE
&#P# )#U# *## ,#+#
 !/$-E*- !&00(
Câu 78: ,YR6F3[??SE*[??SBAEL
Trang 6/59 – Các dạng bài tập Hóa học ôn thi ĐH, CĐ từ 2007 đến 2011
&#BE*A
"
A*5E*A
"
7

# )#BE*A
"

XA5AE7

#
*#BE*A
"
*5E*A
"
7

&

A
"
# ,#5AE7

&

A
"
*5E*A
"
7

#
 !/$*-001
Câu 79: /F??SLB

*A
"
d**E

"
*AABBE
U
*BE$A
U
*
P
E

ABn??SE
€+
&#B

*A
"
*
P
E

AB*E
"
*AAB# )#B

*A
"
BE
U
*d*#
*#BE
U

**E
"
*AABBE$A
U
# ,#d**
P
E

AB*E
"
*AAB#
 !/$*-00+
Câu 80: *F??S`RILB

*A
"

5'7E

$A
U

57E*5"7dBA
"

5U7#}!FSEF??S
IK2m>:!.V[F!\!L
&#5"7575U75'7# )#5U75'7575"7# *#5'7575"75U7# ,#575"75U75'7# !/$*-00(
Câu 81: -F66C$


896K2A

[IIK2 Z#E3•:'Z??=
)5AE7

0'dAE0'IK2??4'+6 :#*4??BAE3>3!%6 :#}!F
S6
&#" )#'P *#'(0 ,#U05-E)0'07
Câu 82L??>!y6!000+EM"#d:]IG @I^_
&#d!Y'0J??TIK2??EMU#
)#-I!%!>!y6!{!\6 !6??E*#
*#d!†??TII!%!>!y6!V#
,#-I!%!>!y6!??'U1W#5-E)0'07
- Hỗn hợp axit td hỗn hợp bazơ. Pt ion thu gọn. Bt điện tích
Câu 83: *U\=L
5'7CbE*→C*


bE

57BAEb5BE
U
7

$A
U

→B

$A

U

bBE
"

bE

A
5"7)*


bB

*A
"

→)*A
"

bB*
5U7BE
"

bE

AbC$A
U


C5AE7



b5BE
U
7

$A
U
*F\=!\=>!c8s
&#5'757# )#575U7# *#5"75U7# ,#575"7# !/$-E*- !)00+
Câu 84: *F\=;L
5'75BE
U
7

$A
U

b)*


x 57*$A
U

b)5BA
"
7

x
5"7B


$A
U

b)*


x 5U7E

$A
U

b)$A
"
x
575BE
U
7

$A
U

b)5AE7


x 5P7C

5$A
U
7

"

b)5BA
"
7


x
*F\=I.`6KT!^;L
&#5'7575"75P7# )#5'75"7575P7# *#575"75U75P7# ,#5"75U7575P7# !/$-E*- !)001
Câu 85: /‡e6??SBAE00'H!e6??SE*00"IK2e6??S4#,?S4
E
&## )#"# *#'# ,#U# !/$-E*- !&00(
Câu 86:
/ '006 ? ?S 5R6)5AE7


0' BAE 0'7H! U00 6 ? ?S 5R6

E

$A
U

00"+
E*00'7IK2??S#}!FSE??S
&#'# )## *#+# ,#P# !/$-E*- !)00+
Câu 87: / '00 6 ? ?S v 2 R6 E

$A

U

00  E* 0' H! '00 6 ? ?S v2R6BAE
0)5AE7


0'IK2??S#,?SE
&#'"0# )#'# *#'0# ,#'(# !/$-E*- !)001
Câu 88: /'006??SEM'R6E*EBA
"

H!'006??SBAERI56t7IK200
6??SEM'#}!FS
&#0"0# )#0'# *#0'# ,#00"# !/$-E*- !)00(
Câu 89: * ? ?S = 0' 6 5BE
U
7

*A
"

F ?• H! ? ?S = "U 6 )5AE7

#$\=
IK266 :#}!FS6
&#'1+# )#"1U# *#'+'# ,#'# !/$*-001
Câu 90: ??S=006*
b
00"6d
b

>6*
c
6
c
U
$A
#/ !K2F6!?
?SU"6#}!FS>JK2L
&#00'00"# )#0000'# *#00"00# ,#0000# !/$*-00+
Câu 91:
Ev2R60'6C$


6*

$>!EBA
"

5[I7
IK2??S
5<=!6!y7 Z?3BA#}!FS
&#00U#)#00+# *#0'#,#00P# !/$-E*- !&00+
Câu 92: Ev2=B

ABE
U
*BE*A
"

)*



66v!3I.89#*v2E

A
5?K7I??SIK2=
&#B*# )#B*BAE)*

#
*#B*BAE# ,#B*BE*A
"
BE
U
*)*

#
 !/$-E*- !)00+
Trang 7/59 – Các dạng bài tập Hóa học ôn thi ĐH, CĐ từ 2007 đến 2011
Câu 93:
/F??SLEBA
"
B*B

$A
U
*5AE7

dE$A
U
5BA

"
7

?YR6F3

I.F?•IK2
H!??S)5E*A
"
7


L
&#EBA
"
*5AE7

dE$A
U
5BA
"
7

# )#B*B

$A
U
*5AE7

#
*#EBA

"
*5AE7

dE$A
U
B

$A
U
# ,#EBA
"
B*B

$A
U
#
 !/$-E*- !)00+
Câu 94: ,?S=F!LC
"b
$A
U
c
BE
U
b
*
c
#*!??S!J89L
cOJ6F?•H!K2?K??SBAEIIK20P+Z Z5hI 7
'0+6 :N

cOJ!F?•H!K2?K??S)*

IK2UPP6 :#
/ !K2F6! IK2 !@??S5uFT@<KH8!7
&#"+"6# )#+0U6# *#+UP6# ,#"6# !/$*-00(
Câu 95:
d!a!I>! !6!5AE7


896K2[I??SE

$A
U

0WIK2?
?S6!
RI+'W#d!6!
&#*# )#X# *## ,#C#
 !/$*-00+
Câu 96:*??SR6L000+6B
b
N000"6*
b
N000P6*
c
N000P
"
E*A

000'6

"
BA

#-q!
8~:*
b
J6K2[I??S=6*5AE7

}ZS
&#0 )#0'0 *#0UUU ,#0'(05-E&0'07
Câu 97: ??=F!L*
b
B
b

"
E*A


*

I6!
*

0'#*'t??\=H!??
BAE5?K7IK26 :#*'t??a!\=H!??*5AE7

5?K7IK2"6 :#w
F:I@!I:??TIK2663m #}!FS6
&#1' )#1P *#(+1 ,#+U+5-E)0'07

B. HÓA HỌC VÔ CƠ
4. Phi kim (halogen – oxi, lưu huỳnh – cacbon, silic – nitơ, photpho): 2
- Điều chế, nhận biết, tính chất hóa học
Câu 98 LEv2 ZR6B

E

< !H!E89'(#-6|!!8T Z58C
6>^F7IK2v2 Z4< !H!E89#E!%3\=2BE
"

&#0W )#"PW *#U0W ,#W
Câu 99:*3IK2?`Iqzm!38!3@!%
&#*A

# )#$A

# *#B

A# ,#BA

#5-E&0'07
Câu 100:*0UU(Z ZBE
"
5I 7I!u=IQ'P6*AIK23m5!\\=
>\7#OJV6 !K2*
&#'"+W# )#(+P"W# *#'U'W# ,#(((W#5-E&0'07
Câu 101: /aZ!%6K|!K|I!.:89F
&#I!%G\B*#
)#??SE*IwF?•H!A


I#
*#I!%G??SB*6V#
,#C


Iz*


 ~!??SB*# !/$-E*- !&00+
Câu 102: /aZ!%6K|!I!.:>!89F
&#I!%GKH#)#!%G*5BA
"
7

#
*#!%Gd*A"

>^FA#,#K3GI @ Z~#
 !/$-E*- !&00(
Câu 103: ˆ?•IG @\!s_
&#/zm!8?JV# )#*nGV#
*#-!.:>!aZ!%6# ,#$F`KH!# !/$-E*- !)001
Câu 104: /aZ!%6IqI!.:6K2~ Z! !:K|!I??S6!!!
8Y#dZ
&#BA# )#BA

# *#B

A# ,#B


# !/$-E*- !&00+
Câu 105: /aZ!%6K|!K|I!.:EBA
"

[
&#BBA"

E$AUIw# )#BBA

E$AU

Iw#
*#BE"

A# ,#BBA"

E*Iw#
 !/$-E*- !)00+
Câu 106: **??SE

$A
U

YF?•H!356!G8;73F Z @
6G @ Z#w F !F?•H!??SBAET Z6`! !F#*3
&#6# )## *#!!# ,#6!!# !/$-E*- !&00(
Câu 107: OG8IG6VII3_
&#d*# )#BE
U

BA
"
# *#BBA
"
# ,#d

*A
"
#
 !/$-E*- !)001
Câu 108: /JZuw!
&#*EOA
U
# )#*
"
5OA
U
7

# *#*5E

OA
U
7

# ,#BE
U
E

OA

U
# !/$-E*- !)00(
Câu 109:
OF8!qIGI^_
Trang 8/59 – Các dạng bài tập Hóa học ôn thi ĐH, CĐ từ 2007 đến 2011
&#OGG3!F2G?KH!?!!5BA
"
c
7!6!5BE
U
b
7#
)#&6v2F6!5BE
U
7

EOA
U

dBA
"
#
*#OGv2=! !IK2;!GBOd#
,#OG@=5BE
U
7

*A
"
#

 !/$-E*- !&001
Câu 110: OG8! 5BOd7v2
&#5BE
U
7

EOA
U

dBA
"
# )#BE
U
E

OA
U

dBA
"
#
*#5BE
U
7
"
OA
U

dBA
"

# ,#5BE
U
7

EOA
U

BBA
"
#
 !/$*-001
Câu 111:
*F\=L
5'7
0

" 
*5BA 7 →
57
0

U 
BE BA →
5"7
0
(0 *O
" 
BE A+ →
5U7
0


" 
BE *+ →
57
0

U
BE *
→
5P7
0

"
BE *A+ →
*F\=I. ZB


L
&#5'75"75U7# )#5'75757# *#575U75P7# ,#5"7575P7# !/$-E*- !&00(
Câu 112: *F\=L
UE*b
A



→
*


b*




bE

A#
E*bCxC*

bE#
'UE*bd*A+



→
d*b**"

b"*

b+EA#
PE*b&x&*"

b"E#
'PE*bdAU

xd*b*

b*

b(EA#
$\=IE*q!%Z>!

&#"# )#U# *## ,#'# !/$-E*- !&00(
Câu 113: *F\=L
57UE*bO8A


→O8*


b*


bE

A#
587E*bBE
U
E*A
"


BE
U
*b*A


bE

A#
57E*bEBA
"



BA


b*


bE

A#
5?7E*bX

X*


bE

#
$\=IE*q!%Z 
&## )#"# *#'# ,#U# !/$-E*- !)001
Câu 114: *F\=L
5'7A
"
b??Sdg→ 57C

bE

A
0


→
5"7A

bE*Iw
0

→
5U7*

b??SE

$→
*F\=I3L
&#5'7575"7 )#5'75"75U7 *#575"75U7 ,#5'7575U7
!/$-E*- !)00(
Câu 115: O\=!%G @I^L
&#dBA
"

0

→
dBA

bA

)#BE
U
BA



0

→
B

bE

A
*#BE
U
*
0

→
BE
"
bE* ,#BE*A
"

0

→
BAEb*A

 !/$-E*- !)00(
Câu 116: /K|2 @>\\=;
&#"A


bE

$


→
$A

bE

A )#C*

bE

$xC$bE*
*#A
"
bdgbE

AxA

bdAEbg

 ,#*

bBAExB*bB*AbE

A
!/$*-00(
Câu 117: ,YR6F3I.F?•IK2H!??SE*YL

&#dBA
"
**A
"
C5AE7
"
# )#C$)$A
U
dAE#
*#&BA
"
5BE
U
7

*A
"
*$# ,#5E*A
"
7

E*AAB*A#
 !/$-E*- !&001
Câu 118: B:'66v!3L*A*

dA
U
d

*


A
+
A


JK2\=H!K2?K
??SE*
Trang 9/59 – Các dạng bài tập Hóa học ôn thi ĐH, CĐ từ 2007 đến 2011
Iw3K2 Z*


!.3
&#dA
U
# )#A

# *#*A*

# ,#d

*

A
+
#
 !/$-E*- !&001
Câu 119: d! !%G   '0066v!3 Ld*A
"


5>^F A

7 dA
U
dBA
"


&BA
"
#*3
K2A


H3
&#d*A
"
# )#dA
U
# *#dBA
"
# ,#&BA
"
#
 !/$-E*- !)001
Câu 120: *3 ZKH6??S6q6ufZ6I~qIK2?`63
z6#dZ
&#BE
"
# )#*A


# *#$A

#,#A
"
# !/$*-001
Câu 121: $A


@q!%Z F\=H!
&#E$AKH)# )#??SBAEA??SdAU#
*#
A
KH
)
??S
dAU
# ,#??SdAE*AKH
)
# !/$*-00+
Câu 122:!G j=P1PW6!>!I!!Ia!R6F3 @=
#-?!?K„!GG
&#U(W# )#UW# *#"1+PW# ,#U+W#5-E)0'07
Câu 123: *IRqFL
" U
 
H PO
KOH KOH
P O X Y Z
+

+ +
→ → →
*F34XJK2L
&#d
"
OA
U
d

EOA
U
dE

OA
U
)#dE

OA
U
d

EOA
U
d
"
OA
U
*#d
"
OA

U
dE

OA
U
d

EOA
U
,#dE

OA
U
d
"
OA
U
d

EOA
U
5-E)0'07
Câu 124: *!KHI!uI~IK2'P(Zv2 Z5I 7R6*A*A

E

#*8
F?•:H!*A5?K7IK2v23m4#Ea8489??SEBA
"
5Y?K7

IK2(1PZBA5\z6 ?3hI 7#OJV6qZ Z*AL
&#'(UW )#(+W *#'U(W ,#+'W5-E !)0''7
- Halogen, lưu huỳnh
Câu 125: OF8!qIGI^_
A. ,?SBC\=H!??S&BA
"

!&C :#
B. g8F ZH86#
C. CZ>!F:#
D. &>!E)Z>!:>!E*#5/$*-0'07
Câu 126: *00'66!236KHIK2006??S#-q'006??S
J?`006??SBAE0'#OJV6. !K2Kf6
A. "1W# B. "+(PW# C. "PW# D. "1+W#5/$*-0'07
Câu 127: *"'P6dA
U
F?•H!??SE*5?K7 !\=>\T6E*8S
>!F
&#00 )#0'P *#0'0 ,#005*-0''7
Câu 128:dZ IGkhông8S>!8h!H}!c#
&#E*EA# )#E

$# *#*A

# ,#$A

# (CĐ 2011)
Câu 129: OF8!qIGkhông I^_
&#!&g @KH6!&CKH
)#CZ>!F6#

*#/F23!>!Fc'ya>!Fb'b"bb+
,#,?SECIK2$!A

5*-0''7
Câu 130: *F3LKBrS$!A

PB
"
OA
U
FeO, Cu C

A
"
#/F33q>!F8h!
??S>!E

$A
U
IwL
&#U )# *#+ ,#P5*-0''7
Câu 131: Ev2 ZIGkhôngR!h!%IK|_
&#E

C

)#*

A


*#E

$B

,#*AA

5-E&0'07
Câu 132:OF8!qkhôngI^L
&#E!Iy8S>!8h!KHh!%IK|#
)#d!6KTyF?`T8#
*#/3\FI.F>!Lc'b'b"bb+F23#
,#/@!%IK2\>389Fv2uw!Fh'00
0
*
aI!%#5-E&0'07
Câu 133: /P 68 mH! U68 KfR!  5I!. !% @  @ Z7
IK2 v 2 m # *  F ?• H! K2 ?K ? ?S E* !\!  v2 Za!6J
@}#-qIF}J[IeZ ZA

5I 7#}!FSe
&#(0#)#""P#*#"0(#,#UU(# !/$*-00(
Câu 134: *'"UUZ Z5hI 7I!uZ??SdAEh'00

*#$ !\=>\
IK2"+6d*#,?SdAERI
Trang 10/59 – Các dạng bài tập Hóa học ôn thi ĐH, CĐ từ 2007 đến 2011
&#0U# )#0# *#0U# ,#0U(# !/$-E*- !)00+
Câu 135: *??S=P0"6v2R6!6!BB454!Q!h
! T!!:6egg&!%X



lX
4
7??S&BA
"
5?K7IK2(P'6 :#
OJV6 !K2Bv28IJ
&#(W# )#U'(W# *#(W# ,#U+W# !/$-E*- !)001
Câu 136:OF8!qIG!_
&#)F ZH8F Zy#
)#-G6I!%86HIG6I!%!#
*#/Z>!EC6Z>!E*#
,#/Z !)
c
HZ !*
c
#5-E !&0''7
Câu 137: B!%GU"(6v2R6d*A
"
dA
U
IK2A

663mR6d

A
U
A

d*#/8K2A


F?•:H!8I~IK20(1PZv2 Z45I 7< !H!E


'P#/JW !K2dA
U
L
&#P+PW )#+U1W *#+0PW ,#+1UW5-E !)0''7
-
c
"
BA
trong H
+
, nhiệt phân của muối nitrat
Câu 138: d!*F?•H!??S=E

$A
U

Y
BBA"
!a
BBA"

\=
&#3>!F# )#6@!K|# *#3 # ,#3>^F# !/$-E*- !)00+
Câu 139: /Q!%!Z!%6L
'7*"(U6*\=H!(06??SEBA
"


'Fe
'

ZBA#
7*"(U6*\=H!(06??S=
EBA
"
'
E

$A
U

0Fe
ZBA#
)!:BA\z6 ?3FqZ ZIh`I!. !%#‚%!ne
'


e


&#
e

M'e
'
# )#
e


Me
'
# *#
e

Me
'
# ,#
e

Me
'
# !/$-E*- !)00+
Câu 140: *" 68*F ?•H! '006 ??S v2R6 EBA
"
0(E

$A
U
0#$ !F
\=>\!eZ ZBA5\z6 ?3hI 7#}!FSe
&#0+UP# )#0P+# *#0UU(# ,#'+1# !/$-E*- !&00(
Câu 141: *668C(006??Sv2R6*5BA
"
7


0E


$A
U

0#$ !F\
=>\IK20P66v28 !6!eZ ZBA5\z6 ?3hI 7#}!FS
6eJK2
&#'0(UU(#)#'0(U# *#'+(U# ,#'+(UU(# !/$-E*- !)001
Câu 142: *v2R6 '' 6C'1 6* U00 6 ??S= v2R6E

$A
U

0
BBA
"

0#$ !F\=>\IK2??S ZBA5\z6 ?37#*
e6??SBAE'??STK2 :IK2H3#}!FS!!qe
&#"P0# )#U0# *#U00# ,#'0# !/$-E*- !&001
Câu 143: BP(6*5BA
"
7


8T Z @= @ Z6|!!IK2U1P63m
v2 Z#E3•KHIqIK2"006??S4#,?S4E89
&#U# )## *#'# ,#"# !/$-E*- !&001
Câu 144: B!%G"UP6v2R6dBA
"


*5BA
"
7

IK2v2 Z
5< !
H! Z!I89'((7#d!K2*5BA"7

v28IJ
&#(P06# )#006# *#''(6# ,#1U06# !/$*-00(
Câu 145: $\z6\=!%G&BA
"
L
A. &BA

A

# B. &

ABA

A

# C. &

ABAA

# D. &BAA

#5/$*-0'07

Câu 146: *6C'006??Sv2R6EBA
"

0(*5BA
"
7


'#$ !F\=>\
IK2016v2 !6! ZBA5\z6 ?37#}!FS
A. ''0# B. ''# C. (U# D. P#5/$*-0'07
Câu 147: /?`IqG8!%??SBE
U
BA
"
H!??S5BE
U
7

$A
U

A. !6!*??SE*# B. ??SBAE??SE*#
C. IR5gg7>!??SE*# D. IR5gg7>!??SBAE#5/$*-0'07
Câu 148: -q]!BA
"
c
??S)5BA
"
7


K|!I‰??SIH!L
&#??SE

$A
U
Y )# !6!*??SB

$A
U
*# !6!*??SE

$A
U
Y ,# !6!* 5*-0''7
Câu 149: *0"68*0P6C5BA
"
7

??=016E

$A
U
5Y7#$ !F\=>\
IK2eZ ZBA5\z6 ?3hI 7#}!FSe
&#P+ )#(1P *#UU( ,#'00(5-E)0'07
Câu 150:*0(+6R6C*&8TIQ"006??E

$A
U

0'#$ !F\=>\
IK20"63mUU(6 Z5I 7F#/6!:8T0U6BBA
"
 !F\=
:^TqZ ZBA5I \z6 ?37 !K26!??
&#0UZ"+06# )#0''Z"+06#
*#0''Z"(P6# ,#0UZ"(P6#5-E !&0''7
Trang 11/59 – Các dạng bài tập Hóa học ôn thi ĐH, CĐ từ 2007 đến 2011
Câu 151:*+P(6*006??R6EBA
"
0PE

$A
U
0#$ !F\=>\5\
z6 ?3BA7@z]8??OŠT !K26! IK2
&#0'P6# )#'1+P6# *#'106# ,#P6#5-E !&
0''7
Câu 152 : Ev2R6C5BA
"
7

*5BA
"
7

&BA
"
#/JW !K2!''(PUW#*q
I!.:IK2!I8!6v28 !6!['U'P6_

&#'0P6 )#+P(6 *#""P6 ,#P+65-E !)0''7
Câu 153: B!%G6K2&BA
"
IK23mv2 Z4#,‡846K2?KE

A
IK2??SX#*8X<6JF ZBA5\z6 ?37#)!:F\
=>\#OJV6 !K2IY\=
&# )#P0W *#+0W ,#+W5-E !)0''7
- Pư tạo NH
4
NO
3
Câu 154: E6K28X6??S>!#$\=IK2??S4 ZX#
B~[[??SBAE5?K74IIK2 Z @6/#&>!
&#E

$A
U

Iw# )#EBA
"
# *#E
"
OA
U
# ,#E

$A
U


Y# !/$*-001
Câu 155: Ea'"006X??SEBA
"
Y?KIK2??S0UU(Z ZB


5I 7#d!K26!??S
A. '(106 )#"+(06 *#"1(06 ,#("6 (CĐ 2011)
Câu 156: *'P6F?•H!??SEBA
"

5?K7#$ !\=>\IK20(1PZ
ZBA5hI 7??S#d!K26! IK2 !68!??S
&#'""6# )#P6# *#(((6# ,#'"16# !/$-E*- !)00(
Câu 157: E'U6&89??SEBA
"

Y5?K7IK2??S
'"UUZ5hI 7
v2 Z4R6! ZB

AB

#/< !v2 Z4H! ZE

'(#*@??SIK2
663m #}!FS6
&#"("U# )#"U0(# *#'0P"(# ,#1+1(# !/$-E*- !&001
5. Đại cương về kim loại: 2

Dãy thế điện cực chuẩn: 1
- Tc vật lí, hóa học, dãy thế điện cực chuẩn
Câu 158:
,YF ! >:!.!\6?J Z >!F 58!:?Y I!%w C"btCbI=KH
w&bt&7L
&#&b*bC"bCb )#C"bCb*b&b
*#&bC"b*bCb ,#C"b&b*bCb
 !/$-E*- !&00+
Câu 159: *F\=>\IGL
5'7 &BA
"

bC5BA
"
7


xC5BA
"
7
"

b&‹
57 bE*x*

bEŒ
,YF!IK2m>:!.V?JZ>!F
&#&bbEbC"b# )#bEb&bC"b#
*#
&b


C"b

Eb

b
# ,#
b

Eb

C"b

&b
# !/$-E*- !)00+
Câu 160:
*F! !6!LXb$bB!bCbO8b#/=QZ>!F!\6?J
&#O8b

€$b

€Cb€B!b

€Xb# )#O8b

€$b

€B!b

€Cb€Xb#

*#
$b

€B!b

€Xb€

O8b

€Cb
# ,#
Xb€$b

€B!b

€Cb€O8b
# !/$*-00+
Câu 161: *\=;LCb*$A
U

xC$A
U

b*#
/\=>\
&#Q Cb

Q>!*# )#Q Cb

Q *b#

*#Q>!CQ>!*# ,#Q>!CQ *b#
 !/$*-00(
Câu 162: E! !6!4F??S6!^F\=;L
b4*
"

x*


b4*

N
4b*


x4*


b#
OF8!qI^L
&#g4b

Z>!6!b#
)#d!6! IK2!
4b
#
*#d!6!Z 6 !6!4#
,#g4
"b


Z>!6!
b
# !/$*-00(
Câu 163: /=Q6w>!Fc ?YI!%FKL
Cb
CN
*b
t*N
C"b
t
Cb
#
*w3 @\=H!
&#*??SC*
"
# )#??SC*


??S**

#
*#C??S**

# ,#C??SC*
"
#
 !/$*-00+
Câu 164: *w3 @>\\=F;
&#*b??S
C*

"
# )#Cb??SE*#
Trang 12/59 – Các dạng bài tập Hóa học ôn thi ĐH, CĐ từ 2007 đến 2011
*#Cb??SC*
"
# ,#*b??SC*#
 !/$*-00(
Câu 165: -q !
C"b

??S!
Cb

q?`6K2?K
&# !6!*# )# !6!&# *# !6!)# ,# !6!#
 !/$*-00+
Câu 166:
%I. @I^L
&#Cb

>!FIK2*#
)#C IK2*b

??S#
*#C"b

Z>!6*b#
,#/Z>!F!V=QLCbEb*b&b#
 !/$-E*- !&00+
Câu 167:

-q !
*b

??S*$A
U

q?` !6!
&#d# )#B# *#C# ,#)# !/$*-00+
Câu 168:   !6 ! \ = IK2 H! ? ?S E

$A
U

Y 4  !6 ! F ?• IK2 H!? ?S
C5BA
"
7
"
#E! !6!4JK258!:=Q?Y:I!%FL
C"b
t
Cb

I=KH
&
b
t&7
&#&# )#*C *#C*# ,#&# !/$-E*- !&00(
Câu 169: /= Q 6  w >! F c   ?Y I!% F K L 
b

tN C
b
tCN *
b
t*NC
"b
tC
b
N
&
b
t&#,Y<R6F3!F?•IK2H!!C
"b

??SL
&#C*&
b
#)#C
b
&# *#**
b
# ,#C*#
 !/$*-001
Câu 170: ,Y  IG<R6F 3 [F?• IK2H! ??S E* [F ?•IK2H!??S
&BA
"
_
&#CB!$# )#&C*A# *#X*# ,#EB*# !/$*-001
Câu 171: d!6!\=IK2H!L??SE*??S*5BA
"

7

??SEBA
"

5Iw!7#d!6!

&#&# )#X# *#C# ,#&# !/$*-00(
Câu 172: *8!:=Q[F!\!Fw>!Fc ?YI!%F5?Y:I!%Qz7KL
X
b
tXNC
b
tCN*
b
t*NC
"b
tC
b
N&
b
t&#
*F !6!!I.\=IK2H!!C
b

??SL
A. X&
b
# B. X*
b

# C. &*
b
# D. &C
"b
#5/$*-0'07
Câu 173: ,YR6F !6!I.F?•IK2H!??SE*K @F?•H!??SEBA
"
Iw
!L
&#C&* )#*C& *#C& ,#*O8&5*-0''7
Câu 174:,YR6F!I.>!IK2 !6!C
&#*
b
&
"b
C
"b
# )#C
"b
*
b
&
b
# *#X
b
*
b
&
b
# ,#*

b
*
b
&
b
# (CĐ 2011)
Câu 175: *Fw3H!<%6K=KL
57C
"
A
U
*5'L'7 587$X5L'7 57X*5'L'7
5?7C

5$A
U
7
"
*5'L'7 57C*

*5L'7 57C*
"
*5'L'7
$w36K2?K??SE*Y
&#U )# *#" ,#5-E)0'07
Câu 176: OF8!qIGkhôngI^_
&#/6@!K| !.66!*5ggg7Z 8SF3>!F6q6!*5eg7#
)#,O8
b
tO8I=KHE

b
tE

?YI!%FO8?•?\=H!??E*Y!!\!
ZE

#
*#*A !F?•H!BE
"
w*AI.IK2*
,#& @\=H!??E

$A
U
YK\=H!??E

$A
U
Iw#5-E)0'07
Câu 177:*F\=L
CbC5BA
"
7
"
"C5BA
"
7

&BA
"

bC5BA
"
7

C5BA
"
7
"
b&#
,Ym>:=QV?JZ>!F! !6!L
&#&
b
C
b
C
"b
 )#C
b
C
"b
&
b 
*#C
b
&
b
C
"b
,#&
b

C
"b
C
b
5-E !&0''7
- Ăn mòn điện hóa, pin điện
Câu 178: *Fw !6 ! 3!:>^Q!:H!LCO8N CXN C $NCB!# d!
^Fw !6!??S>!w !6!IC8SFŽKH
&#'# )## *#U# ,#"# !/$*-00+
Câu 179: *F2 !6L*cC5g7NXcC5gg7NCc*5ggg7N$cC5ge7#d!!:>^H!??S3I!%!T
F2 !66ICI.8SV6aKHL
&#gggge# )#gggggg# *#ggggge# ,#gggggge# !/$-E*- !&001
Câu 180: )!:9!
O8b

??S>!IK2$#d!^! !6!O8$IK2!H!
89?G?‡I!%6??S3I!%!T
&#<O88SV6aI!%F# )#<$8SV6aI!%F#
*#\O8$I. @8SV6aI!%F# ,#\O8$I.8SV6aI!%F# !/$-E*- !&00(
Trang 13/59 – Các dạng bài tập Hóa học ôn thi ĐH, CĐ từ 2007 đến 2011
Câu 181: *U??S!8!%L7E*87
**

7
C*
"
?7E*‡
**

#B^6v!??S6

C3#$K|2>3!%V6aI!%F
&#'# )## *#0# ,#"# !/$-E*- !)00+
Câu 182: /!:8Z!%6L
c/Z!%6'LB^C??S
C*
"
N
c/Z!%6LB^C??S*$A
U
N
c/Z!%6"LB^*??SC*
"
N
c/Z!%6UL*C!:>^H!*R!^??SE*#$K|2>3!%V6aI!%
F
&#"# )#U# *#'# ,##
 !/$-E*- !)00(
Câu 183: /!I!%Xc*uFT !
&#* x*b

b# )#X xX
b

b#
*#Xb

b xX# ,#*b

b x*#
!/$-E*- !)00+

Câu 184: *8!:\=>!Fc >\!I!%FCƒ*L
Cb
*b

x
Cb

b*N•
0

5
Cb
tC7Mƒ0UUe•
0

5
*b
t*7Mb0"Ue#
$3I!%Iz!I!%FCc*
&#'PPe# )#0'0e# *#0+(e# ,#01e#
!/$*-00(
Câu 185: * 3 I!% I z •

 
 F ! I!% FL •

5*c7 M 0UPeN •

54c*7 M ''eN•


5Xc*7 M
0U+e54X8 !6!7#,YF !6!>:!.V?JZ [F!\!
&#4X*# )#X4*# *#*X4#,#*4X# !/$-E*- !)00(
Câu 186: *F:I!%QzL
"b

& t&

Mc'PPeN
b

X tX

Mc0+PeN
b

O8 tO8

Mc0'"eN
b

* t*

Mb0"Ue#/F
!IG!3I!%IH3L
&#O!Xƒ*# )#O!XƒO8# *#O!&ƒX#,#O!O8ƒ*# !/$-E*- !)001
Câu 187: *3I!%IzF!I!%FLXc*''eN*c
&
0UPe#)!::I!%Qz
b


X tX


b

* t*

!FSJK2L
&#c0+Peb0"Ue# )#c'UPec0"Ue#
*#b'Peb0PUe# ,#c'Peb0PUe#
 !/$-E*- !&001
Câu 188: !I!%FI!%QX^??SX$A
U

I!%Q*^??S*$A
U
#
$6|!!!II!%T !K2
&#I!%QX!\6a !K2I!%Q*V#
)#\!I!%QX*I.V#
*#I!%QXVa !K2I!%Q*!\6#
,#\!I!%QX*I.!\6# !/$-E*- !&00(
Câu 189: *8!:L
O!I!%F3I!%Iz89'P'eIK238h!!w>!Fc 
A. O8
b
tO8*
b
t*# B. X

b
tX*
b
t*#
C. 
b
tX
b
tX# D. X
b
tXO8
b
tO8#5/$*-0'07
Câu 190: B:]6892 !6CcX8SV6aI!%FTuFTV6a
&# {6I!a8S>!F )#mI!a8S>!F
*# {6I!a8S>!F ,#mI!a!E
b
8S>!F
Câu 191: *!FS:I!%Qz6w>!c L
Cặp oxi hóa/ khử

M
M
+

X
X
+

Y

Y
+

Z
Z
+
E
0
(V) -2,37 -0,76 -0,13 +0,34
O\=IG>\_
&#bX
b
→
b
bX )#b
b
→
b
b
*#Xb4
b
→X
b
b4  ,#Xb
b
→X
b
b (CĐ 2011)
Câu 192: /uFTI!I!%Xƒ*T
&# !K2I!%QXV )#RI!*

b
??SV
*#RI!X
b
??SV ,# !K2I!%Q*!\6(ĐH khối B 2011)
Trang 14/59 – Các dạng bài tập Hóa học ôn thi ĐH, CĐ từ 2007 đến 2011
- Điện phân, điều chế, tinh chế
Câu 193: BmIK2?`IqI!.: !6!
&#23=! !6!F?•H!3 #
)#>!F! !6!23 !6!#
*# ! !6!23 !6!#
,#23=! !6!F?•H!3>!F# !/$*-001
Câu 194: ,YR6F !6!IK2I!.:@!%89KFI!%G23\
^L
&#B*&#)#B*X#*#B*&#,#C*&# !/$-E*- !&00+
Câu 195: E! !6!qIK2I!.:89KFI!%G??S
&#&# )#BC# *#*&# ,#X#
!/$*-00(
Câu 196: ,YF !6!I.qIK2I!.:89KFI!%G??S6!^L
&#C*&# )#X*# *#&C*#,#)&&# !/$-E*- !&001
Câu 197:
d!I!%GB*\5I!%Q7!@>\
&#Q>!F!*
c
)#Q>!F!B
b
#*#Q !*
c
#,#Q !B
b

# !/$-E*- !&00(
Câu 198: -!%G\&

A
"

H!T5!%3I!%G'00W7IK26 &hP+6
"
5hI 7v2 Z< !H!!I89'P#D3UZ5hI 7v2 Z•??SKH@!
5?K7IK26 :#}!FS6
&#'0(0# )#+P# *#U0# ,#P+# !/$-E*- !)001
Câu 199: -!%G??S=6*$A
U

86B*5H!I!%Q6V7#-q??SI!%
G6!q6RTI!. !%8

&#8l# )#8M# *#8€# ,#8M# !/$-E*- !)00+
Câu 200:
-!%G??S**


H!I!%Q6|!!IK20"6*h@
6K2 Z
h@#E3•K2 Z006??SBAE5h!%IK|7#$\=RI
BAEa!005!\!:qZ??S @I!7#BRI8IJ??SBAE5*M
PU7
&#0'# )#0# *#0'#,#00# !/$-E*- !&00+
Câu 201: -!% G  6 V 00 6 ? ?S = v 2 R6 **



0'  B* 05I!%Q
!%3I!%G'00W7H!K|I?aI!%&"(P0!G#,?SIK2I!%G \V
66&#}!FSH36
&#U0# )#+0# *#'"# ,#U0# !/$-E*- !)001
Câu 202: d!6!qIK2I!.:89F !>!8h! ZE

h!%I#w F !6
! IK2!E
b
??S>!YE

#d!6!
A. # B. &# C. *# D. C#5/$*-0'07
Câu 203: -!%G??S*$A
U

H!89IR57I!%G??S*$A
U

H!89!
5I!%Q7I.IwI!q6
A. h>\Q L*
b

bx*#
B. h>\Q>!FLE

AbxAE
ƒ


bE

#
C. h>\Q>!FL**
b

b#
D. h>\Q LE

AxA


bUE
b

bU#5/$*-0'07
Câu 204: -!%G006??S*$A
U
A5I!%Q7I: !hIK2"6 !6!Tq
Z Z5I 7IK2hL 
&#""PZ )#''Z *#0PZ ,#UZ5*-0''7
Câu 205:O\=I!%G??S**

5H!I!%Q7\=V6aI!%>\ !^2 !6
Xc*??SE*IwI!q6L
&#O\=>\@ •6QF!?aI!%#
)# !*hQG6#
*#O\=hQG6Q6! !6!w! !6!#
,#O\=hQ?KI.Q>!*

c
#5-E&0'07
Câu 206:-!%G5H!I!%Q76??SR6B**$A
U
`6I: !h>3!%8;
ZT?[I!%G#/\uFTI!%G\z6IK2h
&# Z*

A

# )# ZE

A

# *#< Z*

# ,# Z*

E

#5-E&0'07
Câu 207:-!%G5I!%Q7??S=06*$A
U
0'6B*89?aI!%K|I&#
/qZ Z5I 7Fh1P0!GI!%G
&#U0Z# )#1'Z# *#'+1Z# ,#'"UUZ#5-E&0'07
Câu 208: -!%G5H!I!%Q7006??*$A
U
RI>6t6|!!IK2??4‡a6
> !K2!\6(H!??8IJ#*'P(8C4 !F\=>\IK2

'U !6!#}!FS>
&# )#' *#' ,#"5-E)0'07
Trang 15/59 – Các dạng bài tập Hóa học ôn thi ĐH, CĐ từ 2007 đến 2011
Câu 209:Ea'"P(66!$A
U
KHIK2??S#-!%G5H!I!%QK|I?aI!%
@I!7|!!!GIK26 !6!?3h00"6 Zh#*a:|!!I!%
G!GT6 ZIK2h\!I!%Q0'U6#}!FS
&#UU(0# )#"10# *#'P(0# ,#U+((#5-E !&0''7
Câu 210:-!%G??SR6+U6d*(6*5BA
"
7

5I!%Q6V>7I: Z !
K2??S!\6I!'0+6T[I!%G5!\!:K2KH8! @IF q7#/3\F3
??SI!%G
&#dBA
"
dAE# )#dBA
"
d*dAE#
*dBA
"
*5BA
"
7

# ,#dBA
"
EBA

"
*5BA
"
7

#5-E !&0''7
Câu 211:d!I!%G??SB*5QG689mQ?K89T6V>7TL
&#hQ?K>\uF!>!!B
b
hQG6>\uFT !*
c
#
)#hQG6>\uFT E

AhQ?K>\uFT>!*
c
#
*#hQG6>\uFT>!E

AhQ?K>\uFT !*
c
#
,#hQG6>\uFT !B
b
hQ?K>\uFT>!!*
c
#5-E !&0''7
- Kl tác dụng axit HCl, H
2
SO

4 loãng
Câu 212: Ea"6v2R6CX896 K2 [I??SE

$A
U
Y
IK2'"UUZ!I5I 7??S=666!#}!FS6
&#'0+# )#1# *#(1(# ,#+# !/$*-00+
Câu 213: *"P(6v2R6&XF?•H!6K2[I??SE

$A
U

'0W
IK2UZ
ZE


5hI 7#d!K2??SIK2\=
&#'0'P(6#)#((06#*#'0'U(6# ,#1+(06# !/$-E*- !&001
Câu 214: E:++U6 v28&89006??Sv2E*'E

$A
U
0(
IK2??S(+"PZ ZE


5I 7#*@??SIK2K26! 
&#"(1"6# )#'0"(6# *#16# ,#++(P6#

!/$*-00(
Câu 215: *'"6v2F !6!&*CF?•H!K2?K??SE

$A
U

Y5I!.
!% @ @ Z7IK2??S+(UZ ZE5hI 7#
*@??S5I!. !% @
@ Z7IK2666! #}!FS6
&#UP )#U *#U(( ,#U+'
!/$*-00(
Câu 216: *6 6 v 2 &  0 6 ??S = v2 >! E* '  >!E

$A
U
0
IK2"ZE

5hI 7??S45!qZ??S @I!7#,?S4E
&#'# )#P# *#+# ,## !/$-E*- !&00+
Câu 217: *'P+6v2R6! !6!h f!!:6gg&F?•:H!??SE*
5?K7F0P+Z ZE

5hI 7#E! !6!I
&#)# )#*$# *#$)#,#*#
!/$-E*- !)00+
Câu 218: *66v2R6&*??SE*5?K7 ! :^\=!""PZ Z5h
I 7#B:66v26K2?K>!!!5Iw!7 ! :^\=!P+Z
ZBA



5\z6 ?3hI 7#}!FS6
&#'"# )#'P# *#'0# ,#''# !/$-E*- !)00(
Câu 219:  !6!G6Z6gg56gg&7#*'+6v2R6 !6!X
F?•H!K2?K??SE*!0P+Z ZE


5hI 7#w F
!'16F?•H!K2
?K??SE$AUYTqZ Z!I!K
I:''Z5hI 7#d!6!
&#)# )#*# *#$# ,## !/$*-00(
Câu 220: Ev2R6C896K2[I??SE*0WIK2??S
4#BRIC*


??S4'+PW#BRIJV6*

??S4
&#UUW# )#'+PW# *#('W# ,#''+1W# !/$*-00+
- Kl tác dụng HNO
3
, H
2
SO
4

đặc
Câu 221: *v 2 C* \ =H!? ?SEBA

"

Y#$ !\ = IK2??S
<=63 !6!?K#*3I
&#C5BA
"
7
"
# )#C5BA
"
7

# *#EBA
"
#,#*5BA
"
7

# !/$-E*- !)00+
Câu 222:
*v2R6C??S>!E$AUIwI: !F\=
>\
IK2??S46JC @#*3??S4
&#$A
U

C

5$A
U

7
"
# )#
$AU
C

5$A
U
7
"


C$AU
#
*#$AU

C$AU# ,#$AU#
!/$*-00+
Câu 223: /qZ??SEBA
"
'5Y7Z3J?`Iq6v2R60'6C
0'6*58!:\=3 ?3BA7
&#0(Z# )#'0Z# *#0PZ# ,#'Z# !/$-E*- !)00(
Câu 224: * "P 6  F ?• : H! ? ?S EBA
"

5?K7 !  U Z Z  5\ z6 ?3h
I 7#dZ
&#BA# )#BA# *#B# ,#BA#
!/$*-00(

Trang 16/59 – Các dạng bài tập Hóa học ôn thi ĐH, CĐ từ 2007 đến 2011
Câu 225:
Ea'6v2C*5<%6'L'789>!EBA"IK2eZ5I 7v2 Z
5R6BABA7??S45<=!6!>!?K7#/< !

I!H!E

89'1#}!FSe
&#U# )#UU(# *#P0# ,#""P# !/$-E*- !&00+
Câu 226: E((P6v2R6&??SEBA
"

YIK2??S
"'"PZ5hI 7v24R6! Z @6I6 ZG @ Z#d!K2
4'(6#*??SBAE5?K7I @ Z6`! !F#OJV6 !
K2&v28IJ
&#'1"W# )#'(0W# *#'0W# ,#'W# !/$*-001
Câu 227:
*P+6C??S=0"6E$AUIw5!\!:$A\z6
 ?37#
$ !\=>\IK2
&#0'6
C$AU
# )#006C

5$A
U
7
"


00(6
C$AU
#
*#006C5$AU7"

006C?K# ,#00"6C5$AU7"

00P6C$AU#
!/$-E*- !)00+
Câu 228: * "0U 6 6 !6 !   :  ? ?S EBA
"

Y  IK2 1U0( 6 Z
B
>
A


5\
z6 ?3hI 7< !I!H!E


89#dZB
>
A


 !6!
&#BA# )#BA



&# *#B

A&#,#B

AC#
!/$-E*- !&001
Câu 229: *v2R6P+60(6AF?•:H!K2?K??SEBA
"
#$ !F\
=>\IK20(1PZ6 Z5I 7??S4#D68!??S4IK2UP6
6! #dZ
A. B

# B. BA# C. B

A# D. BA

#5/$*-0'07
Câu 230:B"6v2R6F !6!C&X>!6|!!IK2+'6v
24#Ea4??SEBA
"
5?K7IK20P+Z ZBA5\z6 ?3hI 7#$6
EBA
"
IY\=
&#0' # )#0'U# *#0'P# ,#0'(#5-E)0'07
Câu 231:-66v2*C<% !K2K=+L"H!6K2??SEBA
"
#d!F

\= :^IK20+663m??SPZv2 Z5I 7R6BABA

5 @\
z6  FB
b
7#)!:K2EBA
"
IY\=UU'6#}!FS6
&#UU(# )#U0# *#""P# ,#0U#5-E !&0''7
- Kl tác dụng dung dịch muối
Câu 232: *v28&C??S=*5BA
"
7


&BA
"
#$ !F\=>\
IK2v2mR68 !6!L
&#C*&# )#&*&# *#&C*# ,#&C&# !/$*-00(
Câu 233: *v2R6CX??S&BA
"

I: !F\=>\IK2?
?SR6!6!3m4R6! !6!#E!6!
&#C5BA
"
7
"


X5BA
"
7

# )#X5BA
"
7


C5BA
"
7

#
*#&BA
"

X5BA
"
7

# ,#C5BA
"
7


&BA
"
#
!/$-E*- !&001

Câu 234: *v28R6+6&P6C06??S&BA
"
'#$ !F \=
>\IK2663m#}!FS658!:=Q?Y:I!%FL
C"b
t
Cb

I=KH
&
b
t&7
&#"U# )#PU(# *#1U# ,#U0#
!/$-E*- !&00(
Câu 235: *6
'

6& '006??SR6*5BA
"
7


0"&BA
"

0"#$ !F\=>\
TIK26


63m#B:6



6F?•H!K2?K??SE*TIK2
0""PZ Z5hI 7#}!FS6
'

6


JK2
&#('0U"# )#'0(U"# *#0U'P# ,#'0('P# !/$*-001
Câu 236: /!:!Z!%6L
c/Z!%6'L*668C5?K7e
'

Z??S*5BA
"
7


'N
c/Z!%6L*668C5?K7e

Z??S&BA
"

0'#
$ !F\=>\ !K23mIK2h!Z!%6I.89#}!FSe
'
H!

e


&#
e
'

M'0e# )#
e
'

Me# *#
e
'

Me#,#
e
'

Me#
!/$-E*- !)00(
Câu 237: *6K28X??SR6C*


**

#d!K23m !F\=>\
~ !K28X8IJ06#*@J??S\=IK2'"P66! #
/ !K2F6!
&#'"'6# )#'U'6# *#'+06#,#'16# !/$-E*- !)00(

Câu 238: *v 2 R6' 6   > 6 X  ? ?S =  6 *
b

 ' 6 &
b
I: !F
\=>\IK26??S=8! !6!#/F!FSIG!FS>\
6YK|2_
&#'(# )#'# *#'# ,#0# !/$-E*- !&001
Trang 17/59 – Các dạng bài tập Hóa học ôn thi ĐH, CĐ từ 2007 đến 2011
Câu 239: *66v28XCK2?K??S*$A
U
#$ ! :^F\= ; 8~
J??SIK2668m#/JJV6 !K2Xv288IJ
&#10+W# )#'P+W# *#("0W# ,#(0W# !/$-E*- !)00+
Câu 240: B^6mw'006'006??Sv2R6*5BA
"
7


0&BA
"

0#
$ 6 |! ! 3  !6 !    6 @ G IK2 '0'+ 6 5!\!:F !6!I.
8F6:m7#d!K2mIY\=
&#'U06# )#'P6# *#0(U6#,#'+6# !/$-E*- !)001
Câu 241: B^6F !6!5<FS!237 !K206006? ?S
&BA
"


'  I: ! \ = >\  # D; ? ?SI6 @   IK2'((66! #d!6
!
&#C# )#*# *## ,#X# !/$*-001
Câu 242 L*'1"6v28X*<%6K='L??S=06C

5$A
U
7
"
#$
!F\=>\IK266 !6!#}!FS6
&#PU0 )#'P" *#'00 ,#'(05-E&0'07
- Kl tác dụng với phi kim
Câu 243: E('6v2R6C

A
"
AXA006>!E

$A
U

0'5[I7# $
\=v26!y IK2 !@??S !K2
&#P('6# )#U('6# *#"('6#,#('6# !/$-E*- !&00+
Câu 244: *'"6v2R68 !6!*&h?8F?•H!>!IK2v
24R6F>! !K2"""6#/qZ??SE*[IIq\=:H!4
&#06# )#+6# *#+6# ,#106# !/$-E*- !&00(
Câu 245: B'P(6v2R6&&*CXH!6K2?K ZA


I: !F\=>\
IK2"63m#/qZ??SE*[IIq\=H!3m
&#U006# )#006# *#(006# ,#P006# !/$*-001
Câu 246: -q     " 6 v 2 R6 CA C
"
A
U
C

A
"
5 I  6 CA
896
CA"7J?`[IeZ??SE*'#}!FSe
&#0'P# )#0'(# *#0"# ,#00(# !/$-E*- !&00(
Câu 247: *1'6v2R6CAC

A
"
C
"
A
U

F?•H!??SE*5?K7#$ !F\=>\
IK2??S4N@4IK2+P6C*


66

C*
"
#}!FS6
&#(+# )#1+# *#P0# ,#+(0# !/$-E*- !)00(
Câu 248: * ''"P 6 v 2 R6 C CA C

A
"
C
"
A
U

\ = : H! ? ?S EBA
"
Y5?K7
IK2'"UUZ ZBA5\z6 ?3hI 7??S#*@??SIK2666! #
}!FS6
&#"0# )#"U"P# *#U101# ,#"(+# !/$-E*- !&00(
Câu 249: -F+6 !6!5FS! @I!237v2 Z*



A

#$\=IK2"063mqZv2 ZIY\=PZ5hI 7#d!6!
&## )#*# *#)# ,#*# !/$*-001
Câu 250: /P68mH!U68KfR!5I!. !% @ @ Z7
IK2 v 2 m # *  F ?• H! K2 ?K ? ?S E* !\!  v2 Za!6J
@}#-qIF}J[IeZ ZA


5I 7#}!FSe
&#(0# )#""P# *#"0(# ,#UU(# !/$*-00(
Câu 251: -F'+U6v2& Z>!5?K7IK2"06v2>!#/qZ Z
>!5I 7IY6!\=L
&#'+1Z )#UU(Z *#''0Z ,#(1PZ5*-0''7
- Phản ứng nhiệt luyện
Câu 252: *R ZE

5?K7uv2F>!*AC

A
"
XAAh!%I#$\=v
2ma!L
&#*CXA# )#*CXAA#
*#*CX# ,#*CAXAA# !/$-E*- !&00+
Câu 253:
* Z*A5?K7I!=IQv2R6&A"AC"AU*A
IK23
m4#*4??SBAE5?K7 3 ‘3a!J @X#}!\F\=>\#
OJ @XR6
&#C*# )#AC
"
A
U
*# *#AC*# ,#&C*# !/$*-00+
Câu 254: ,‡[[eZ Z*A5hI 7I!u6=IQK2?Kv2mR6*AC

A

"

5h!%I
7#$ !F\=>\IK2 Z#,‡8 ZhK2?K??S*5AE7

TU6 :#}!FSe
&#''0# )#0(1P# *#0UU(# ,#0U# !/$*-00(
Câu 255: *eZv2 Z5hI 7R6*AE


\=H!6K2?Kv2mR6*AC
"
A
U
#$ !F\=>\ !K2v2m!\60"6#}!FSe
&#0''# )#0P0# *#0U# ,#0UU(#
!/$-E*- !&00(
Trang 18/59 – Các dạng bài tập Hóa học ôn thi ĐH, CĐ từ 2007 đến 2011
U U
Câu 256: *R Z*A5?K7I!u1'6v2R6*A&

A
"

I: !\=
IK2("63m#d!K2*Av28IJ
&#0(6# )#("6# *#06#,#U06# !/$-E*- !&001
Câu 257: *UU(Z Z*A5hI 7[[I!u=IQ(66>!mI: !\=>\
#dZIK2\=< !H!!I890#*@=>!mJV6qZ Z
*A


v2 Z\=
&#C"AUN+W# )#CA"N+W# *#CA"NPW# ,#CAN+W#
!/$*-00+
6. Kim loại kiềm, kim loại kiềm thổ, nhôm, sắt: 5
- Điều chế, tinh chế
Câu 258: /@!%!!I>!IK2\>389KF
&#I!%G??S
BBA"
 @6VI!%Q#
)#I!%G??SB*6VI!%Q#
*#I!%G??SB* @6VI!%Q#
,#I!%GB*\# !/$*-00+
Câu 259: /Q!%FZ!%6L
5g7*??SB*??SdAE#
5gg7*??SB

*A
"

??S*5AE7

#
5ggg7-!%G??SB*H!I!%Q6V#
5ge7**5AE7


??SBBA
"
#5e7$• ZBE

"

??SB

*A
"
#
5eg7*??SB

$A
U

??S)5AE7

#
*FZ!%6I.I!.:IK2BAEL
&#gggggeg# )#gggggg# *#ggee#,#ggeeg# !/$-E*- !)001
Câu 260: O\=F;>\K|2?KH!IG @!\=!%@6_
&#&F?•H!C
"
A
U
# )#&F?•H!>!E

$A
U
Iw#
*#&F?•H!*A# ,#&F?•H!CA"

#

!/$*-00+
Câu 261:
-qIK2&A"

[v2&A"

CA"K|!JK2L
&#?`??SBAE5?K7 Z*A

5?K7R!#
)#?` ZE

h!%I??SBAE5?K7#
*#?` Z*Ah!%I??SE*5?K7#
,#?`??SBAE5?K7??SE*5?K7R!# !/$-E*- !)00+
Câu 262: Bm%j[L
&#,`*Aw**A
"

Iq 3$!O$’IqIK2j#
)#,`3 *A >!mmh!%I#
*#,`
A

>!FF3$!O$’IqIK2j#
,#/V66K28IqIK2j# !/$-E*- !)00(
Câu 263: *1'66!!I8\=:H!??SE

$A
U


5?K7IK2??S=+6
6!y#*@=6!!I8
A. BE*A
"
# B. *5E*A
"
7

# C. 5E*A
"
7

# D. )5E*A
"
7

#5/$*-0'07
Câu 264: ,YR6F !6!36!q]KG6 !L
A. D!B*# B. Bd# C. )*# D. D!Bd#5/$*-0'07
Câu 265: EPPU6v26!! !6! !.6! T :!:KH
IK2??S#*8??SF?•H!??S&BA
"
5?K7IK2'(P6 :#
E! !6! !.6L
&#Bd )#k8* *#D!B ,#dk85*-0''7
Câu 266: ,YR6F !6!I.36!q]KG6 !L
&#Bd*) )#D!Bdk8 *#D!Bd ,#Bd*)5*-0''7
Câu 267: B[w38T ZL5'7Cb$5757C


A
"
b*A5 75"7&bA

5 75U7*b*5BA
"
7

5757*bdBA
"
575P7&bB*57#*FK|2>\\=>!F !6!L
&#5'75"75P7 )#575"75U7 *#5'75U757 ,#57575P75-E&0'07
Câu 268: OF8!qIGI^_
&#*F !6!L!8!8!I.F?•H!KHh!%IK|
)#d!6!>!IK2?`Iq::8uI!%
*#d!6!6! !q6!q]KG6?!%
,#/!.V?JI!%ZGF !6! !.65[8!I:8!7!%I\!\6?J
5-E&0'07
Câu 269:*+'6v2R66 !6! !.66 !6! !.64F?•:H!K2?K??S
E*YIK2PZ Z5I 7#d!6!4
&#!6!# )#!!8!# *# !>!# ,# !8!#5-E&0'07
Trang 19/59 – Các dạng bài tập Hóa học ôn thi ĐH, CĐ từ 2007 đến 2011
Câu 270:Ea(1U6v2R6Bd)KHIK2??SP((Z ZE

5I 7#,?S4R6E*E

$A
U
<%6K=UL'#/a??S8h!??S4 !
K2F6!IK2

&#'"+06# )#'(UP6# *#'+(6# ,#'UP6#5-E&0'07
Câu 271: OF8!qIGkhông I^_
&#??I]6IwB

$!A
"
d

$!A
"
IK2;!Ž!~
)#-F6F6!qIK2?]m89F @
*#*C

*

8S36?•? !\! ZuqTFŽJs
,#/aZ!%6B

IK2I!.:89FI??BE
U
BA

8Y5-E)0'07
Câu 272: EU6v2R6! !6! !.6006??E*'IK2?
?S4=F3RI689#E! !6!
&#* )#) *#$ ,#)*5-E)0'07
Câu 273LO•IK2?`@!%?@!%!33J666
\!36KH#*@=;•
&#D!


$A
U
#&

5$A
U
7
"
#UE

A# )#d

$A
U
#&

5$A
U
7
"
#UE

A#
*#5BE
U
7

$A
U

#&

5$A
U
7
"
#UE

A# ,#5BE
U
7

$A
U
#&

5$A
U
7
"
#UE

A#5-E !&0''7
Câu 274:E23>!IK2?`IqI^K288 !Y>K_
&#e@!5*A7#
)#/5*$A
U
#E

A7#

*#-F@!5**A
"
7#
,#/5*$A
U
#E

A7#5-E !&0''7
Câu 275:d!K2!>! !6!'t6
"
#}!\!:9!q>!FnTJ
!:6+UWqZ!qJa! v#)F Z>!ZZ:
&#0'6# )#0'(6# *#0'1P6#,#0'P(6#5-E !&0''7
- Tc hóa học, sơ đồ phản ứng
Câu 276: * ?Y F 3LBE
U
* 5BE
U
7

$A
U
 B* *

 C*


&*"
#$3  ?Y F?•H!K2
?K??S)5AE7



 :
&## )#U# *#'# ,#"# !/$*-00(
Câu 277:
*IR\=LB*x57xBE*A
"

x547xBBA
"
#4q
&#BAEB*A# )#BAEB*A"#
*#B*A"B*A# ,#B*A"

B*A"#
!/$*-00+
Câu 278:
/[!6!4Q!%F\=L



→

'
b*A


'
bE


Ax



b4xb4
'
bE

A 

b4xb4

bE

A
E!6!4K=
&#)*A
"

B*A"

)#**A
"

BE$A
U
*#*A
"
BE*A
"

,#**A
"


BE*A
"

 !/$-E*- !&00(
Câu 279: *'P6v2R6&&

A
"
\=:H!??SE*5?K7IK2eZ ZE

5I 7
??S#B~[[??SBE
"
I:?K??SIK2 :;:K2 :I: !
K2 @I!IK20U63m#}!FSe
A. 0UU(# B. 0U# C. '"UU# D. 0P+#5/$*-0'07
Câu 280: *IRqFL
*A
b
→
**

b4
→

*5BA

"
7

bX
→

**A
"
#
*@=4XJK2L
A. E*&BA
"
5BE
U
7

*A
"
# B. *

&BA
"
*A
"
#
C. *

EBA
"
*A


# D. E*EBA
"
B

*A
"
#5/$*-0'07
Câu 281: -qPU6v2R6 !6!k5<FSgg7>!J[IU006?
?SE*'#d!6!kL
&#) )#* *#) ,#5*-0''7
Câu 282:*66BAEZ??SBE*A
"
RI6tIK2Z??S#D3'Z??S
F?•H!??S)*

5?K7IK2''(6 :#w F'Z??S??S**

5?K7
R!I ! :^F\=IK2+06 :#}!FS6K=
&#00UU( )#00+" *#00(U( ,#0'UU5-E&0'07
Câu 283: ,YR6F !6!` !q6!q]KG6 !L
&#Bd) )#*) *#Bd* ,#D!B5-E !)0''7
- Nước cứng
Trang 20/59 – Các dạng bài tập Hóa học ôn thi ĐH, CĐ từ 2007 đến 2011
Câu 284:  6‡ KH = = F !L *
b
 
 b


c
"
E*A
 *
c

c
U
$A
# *3 IK2 ?` Iq 66.6 6‡
KH=
&#E*# )#E$AU# *#B*A"# ,#BE*A"#
!/$-E*- !)00(
Câu 285: E!3IK2?`Iq66.6KH=‘
&#
B*A"

E*# )#
B*A"

B
"
OA
U
#
*#B*A"

*5AE7

# ,#B**5AE7#

!/$*-00(
Câu 286: KH=F!LB
b
50067
b
50067*
b
500U67*
c
50067E*A
"
c
50'0
67$A
U
c
500'67#-@!KHI: !\=>\TKHa!
&#Z=J )#Z=‘
*#KH6.6 ,#Z=6|!5*-0''7
Câu 287:,YR6F3I.q663Z=6|!KHL
&#E*BAEB

*A
"
)#BAEB
"
OA
U
B


*A
"
#
*#d**5AE7

B

*A
"
# ,#E**5AE7

B

*A
"
#5-E !&0''7
- Kl tác dụng với nước, axit, bazơ, muối
Câu 288: *66‡2 !6Bc)F?•H!KH5?K7IK2??S""PZE


5I 7#/qZ
??S>!E$AU

J?`Iq??S
&#P06# )#'06# *#"06# ,#+6# !/$*-00+
Câu 289: d!'006??SdAE''006??SE*IK2??S=P63#
BRI65w6t7E*??SIY?`
&#0+# )#'# *#0# ,#0# !/$*-00+
Câu 290: *!66&!J89L
cOJ6F?•H!K2?K??SBAE!>6 ZE


N
cOJ!F?•H!K2?K??SEBA
"

Y!6 ZB

A5\z6 ?37#‚%!n
>
&#>M# )#M># *#>MU# ,#>M# !/$*-00(
Câu 291: *v2R6B&<%6K='LKH5?K7#$ !F\=>\
IK2(1PZ ZE


5hI 7663m @#}!FS6
&#U# )#+(# *#'0(# ,#U"#
!/$-E*- !&00(
Câu 292: Ev2R6B&#*666K2?KKHTFeZ Z#B:i66
??SBAE5?K7TIK2'+eZ Z#/JJV6 !K2B58!:Fq
Z ZI`I!. !%7
&#++"'W# )#"1(+W# *#U1(+W# ,#1(+W# !/$-E*- !)00+
Câu 293: E16v2R6 !6!>!KHIK2006??S=
63RI00U0UZ ZE


5hI 7#d!6!
&#B# )#*# *#)# ,#d# !/$-E*- !)001
Câu 294: E0"6v2R6&&
U
*

"
??SdAE5?K7IK26v2 Z
??S#$• Z*A


5?K7??SK2 :IK2UP(6#}!FS
&#0# )#0P0# *#0U# ,#0U0# !/$-E*- !&00(
Câu 295: *1(6v28R6XCP006??S*$A
U

0#$ !F\=>\
IK2??S"0U6v2 !6!#OJV6. !K2Cv28IJ
A. "+(W# B. P"+W# C. PUUW# D. U"PW#5/$*-0'07
- CO
2
,
SO2
, P
2
O
5

td dung dịch kiềm
Câu 296: B'"U6v26!8 !6!SIK2P(63m Z#DK2
Z!3•+6??SBAE' !K26! IK2\=
&#(6# )#U6# *#P"6#,#P6# !/$-E*- !)00+
Câu 297: E3•UU(Z Z*A


5I 7006??Sv2R6BAE0'

)5AE7

0
!66 :#}!FS6
&#1(# )#''(# *#'1+0# ,#'++"# !/$-E*- !&00(
Câu 298: *0UU(Z Z*A


5hI 73•:'006??S=v2BAE00P
)5AE7

0'IK266 :#}!FS6
&#''(# )#"1U0# *#'1+0# ,#"PU# !/$-E*- !&001
Câu 299: 
E3•P((Z Z*A5I 7Z??S)5AE7

RI6t
IK2'+P
6 :#}!FS
&#00"# )#00U(# *#00P# ,#00U#
!/$-E*- !&00+
Câu 300: *'006??SdAE'006??SE
"
OA
U

0IK2??S#*@?
?SIK2v2R6F3
&#dE


OA
U

d
"
OA
U
# )#dE

OA
U

d

EOA
U
#
*#dE

OA
U

E
"
OA
U
# ,#d
"
OA
U


dAE#
!/$-E*- !)001
Trang 21/59 – Các dạng bài tập Hóa học ôn thi ĐH, CĐ từ 2007 đến 2011
Câu 301: *0'6O

A


??S=0"6dAE#,?SIK2F3L
&#dEOAUdEOAU# )#d"OAUdAE# *#E"OAUdEOAU# ,#d"OAUdEOAU#
!/$-E*- !)00(
Câu 302: E3•""PZ Z*A


5I 7'6??S)5AE7


'IK2??S#*!q
Z??S @I!RI63??S
A. 0U# B. 0P# C. 0'# D. 0#5/$*-0'07
Câu 303LE3•0P+Z Z*A

5I 7'Z??SR6BAE00*5AE7

00'
IK2>6 :#}!FS>
&#00# )#0+# *#'00# ,#'#5-E !&0''7
Câu 304: E3•UZ*A


5I 7'006??SR6d

*A
"
0dAE>6tZ !F
\=>\IK2??S4#*84F?•H!??S)*

5?K7IK2''(6
:#}!FS>L
&#'0 )#'U *#' ,#'P5-E !)0''7
-
c
"
*A
tác dụng H
+

Câu 305: *[[??S=6E*??S=86
B*A"
IR|! 3I.IK2e
Z Z5hI 7??S#d!?KKH@!??S3>3!% :#)!q=!%
!neH!8L
&#eMU5c87#)#eM''5c87#*#eM''5b87#,#eMU5b87# !/$-E*- !&00+
Câu 306: ,?S=v2R6B

*A
"

'dE*A
"


'#B~[[ [!;I::006
??SE*''006??S!eZ Z5hI 7#}!FSe
&#UU(# )#""P# *#U# ,#''# !/$-E*- !&001
Câu 307: *'16v26!8!I8 !6! !.6F?•:H!??SE*
5?K7!0UU(Z Z5hI 7#d!6!
&#B# )#D!# *#d# ,#k8# !/$-E*- !)00(
Câu 308: B!%GU066!uwI@@6!‡3!(1PZ Z*A

5I 7#/
JJV6. !K2**A
"
#*A
"

!uw
&#0W# )#(UW# *#U0W# ,#1W# !/$-E*- !)00(
Câu 309: B~[[[!;I::"06??SE*''006??S=B

*A
"
0BE*A
"
0\=IK26*A


&#00"0# )#00'0# *#000# ,#00'#5-E&0'07
- Tính lưỡng tính của Al(OH)
3
, Zn(OH)

2
Câu 310: B~[[I:?K??SBAE??S
&*"
#E!%K2>\
&# : mI :# )#< : m#
*# : m Z8# ,# @ : Z8# !/$-E*- !&00+
Câu 311: B~[[0Z??SBAE '0U ??SR600U6C*
"
N00'P6
&

5$A
U
7
"


00U6E

$A
U

IK266 :#}!FS6
&#P(# )#'P0# *#U'(# ,#0PU# !/$*-001
Câu 312: EU+U6•d&5$A
U
7

#'E


AKHIK2??S#*8
F?•H!006??S)5AE7


'\=IK266 :#}!FS6
&#+(# )#UPP# *#UU# ,#P# !/$*-001
Câu 313: /??S=6
&*"
H!??S=86BAE#-qIK2 :TJ<%
&#L8M'LU# )#L8l'LU# *#L8M'L# ,#L8€'LU# !/$-E*- !&
00+
Câu 314: *006??S
&*"
'F?•H!eZ??SBAE0K2 :IK2'P
6#}!FSH3e
&#'# )#'(# *#U# ,## !/$-E*- !)00+
Câu 315: *eZ??SBAE??S=0'6&

5$A
U
7
"

0'6E

$A
U
I: ! \ =
IK2+(6 :#}!FSH3eIqIK2K2 :
&#0"# )#0U# *#0# ,#00# !/$-E*- !&00(

Câu 316: /666 !"006??S=
)5AE7

0'BAE0'IK2??S#*[
[??S006??S&

5$A
U
7
"

0'IK2 :4#-qIK2K2 :4H3T!FS
6
&#'1# )#''+# *#'1# ,#'+'# !/$*-00+
Câu 317: E:66X$A
U

KHIK2??S#*''06??SdAEIK2
6 :#w F:'U06??SdAETiIK26 :#}!FS6
&#0'# )#U0# *#'"+#,#'++'0# !/$-E*- !&001
Câu 318: E66v2R6B

A&

A
"

E

AIK2006??S4<=

3?3RI0#/! Z*A


5?K74IK26 :#}!FS6JK2
Trang 22/59 – Các dạng bài tập Hóa học ôn thi ĐH, CĐ từ 2007 đến 2011
&#("+# )#''"+(# *#'"""1# ,#(+(# !/$*-001
Câu 319: B~[[??SBAEI:?K??S#$ !F\=>\<IK2?
?S#*3??S
A. *$A
U
# B. C5BA
"
7
"
# C. &*
"
# D. *5E*A
"
7

#5/$*-0'07
Câu 320: E66X$A
U
KHIK2??S#B:''06??SdAET
IK2"6 :#w F:'U06??SdAETIK26 :#}!FS6
&#"0 )#U' *#'++' ,#'P'05-E&0'07
Câu 321: *F3LBE*A
"
*A&5AE7
"

C5AE7
"
EC*

BE
U
*#$3F?•IK2H!??SBAE
Yh!%IK|
&#U )# *#" ,#P5-E&0'07
Câu 322: *'06??dAE'F?•H!'006??&*
"
RI>6tIK2??4UP(6 :#
D!8~ :6!:'+6??dAE'4IK2"U6 :#}!FS>
&#' )#0( *#01 ,#'05-E)0'07
Câu 323: OF8!qIGsai?
&#B

*A
"
!%u;@!%\>3!#
)#r!%IK|3\ !6! !.6I.F?•IK2H!KH#
*#B@68.6@!K| @ ZKH?6>!&

A
"
8.n8\%
,#/!.V?JI!%ZG!%I\ !6! !.6!\6?J5-E !)0''7
Câu 324: *U006??S•R6&*
"
>6tZ&


5$A
U
7
"
6tZF?•H!P'6??SBAE'
 !F\= :^IK2(UU6 :#w F !U006•F?•H!??S)*

5?K7
TIK2""6 :#/<%>L
&#UL" )#"LU *#+LU ,#"L5-E !)0''7
Câu 325:*?YF3L$!A

*5AE7
"
*A
"
X5AE7

BE*A
"
&

A
"
#$3?YF?•IK2H!?
?SBAE5Iw7
&#P )#" *# ,#U5-E !)0''7
- Phản ứng nhiệt nhôm
Câu 326: -qI!.:IK2+(6*[*


A
"

5?K789KF!%@6H!!%3\=
10WT !K28@6J?`!!q
&#('06# )#U06# *#U06# ,#U06# !/$*-001
Câu 327: Bv28R6'6*

A
"
66&h!%I#$ !\=IK2
""6v2m#*8v2\=H!>!E*5?K7F
eZ ZE5I 7#}!FSe
&#UU(# )#""P# *#+(U# ,#'00(# !/$-E*- !)00+
Câu 328: d!U'U6v2R6C

A
"
*

A
"

&A"
F?•H!??SBAEIw5?K7\=
IK23m !K2'P6#-q U'U689\=!%@6\!?`'0(6&#/
JJV6 !K2*

A

"


v2
&#0""W# )#0P+W# *#"P+'W# ,#PPP+W# !/$*-00+
Câu 329: B  66 v 2 &  C

A
"
5 6@! K| @  @ Z7 I: !\=>\
IK2v2m4#*!4!J89L
cOJ'F?•H!??SE

$A
U

Y5?K7!"0(Z ZE

5I 7N
cOJF?•H!??SBAE5?K7!0(UZ ZE


5I 7#}!FS6
&#+ )#1U"# *#1U0# ,#'U0#
!/$-E*- !&00(
Câu 330: B66v2R6&C
"
A
U


I!. !% @ @ Z#$ !\=>\
IK2v2m#*F?•H!??SBAE5?K7IK2??S43mX""P
Z ZE


5hI 7#$• Z*A


5?K7??S4IK2
"16 :#}!FS6
&#UP# )#U("# *#"P+# ,#"P+#
 !/$-E*- !)001
Câu 331: -6v2R6&'P6C

A
"

5I!. !% @ @ Z7I: !\=>\
IK2v2m#*F?•[IH!e6??SBAE'!""PZE



5I 7#
}!FSe
&#'0# )#'00# *#00# ,#"00# !/$*-00(
Câu 332: /Q!%F\=!%@6v2R666&UP6*

A
"
5I!. !% @A


7
 !\= :^IK2v2#*86K2?K??SE*5Y7 !F
\=>\IK20'PZE

5I 7#*a:86K2?K??SBAE5Iw
7 !\= :^T6BAEIY\=L
&#00P6 )#0'U6 *#00(6 ,#0'P65-E !)0''7
- Fe áp dụng công thức kinh nghiệm
Câu 333: B668m>!IK2"6v23m#Ea:v2??S
EBA
"
5?K7F0PZ5I 7BA5\z6 ?37#}!FS6
&## )## *#"# ,#P# !/$-E*- !)00+
Câu 334: * ''"P 6 v 2 R6 C CA C

A
"
C
"
A
U
\ = : H! ? ?S EBA
"
Y5?K7
IK2'"UUZ ZBA5\z6 ?3hI 7??S#*@??SIK2666! #
Trang 23/59 – Các dạng bài tập Hóa học ôn thi ĐH, CĐ từ 2007 đến 2011
}!FS6
&#"0# )#"U"P# *#U101# ,#"(+# !/$-E*- !&00(
Câu 335: Bv2R6'0(6&'P06C


A
"
5I!. !% @ @ Z7 !\=>\
IK23m4#d!K2 !6!4L
&#P6 )#U6 *#''6 ,#'PP65*-0''7
Câu 336: ,YR6F>!I.8S& h!%IL
&#CA*A*

A
"
)#O8Ad

A$A*#CAA*A ,#C
"
A
U
$A)A 5*-0''7
Câu 337: /'0(8&H!"U(8C
"
A
U
R!!:\=!%@6I!. !% @ @ Z#
Ev2m\=89??E

$A
U
Y5?K7IK2'0+Z ZE

5I 7#E!%3

\=!%@6
&#(0W )#10W *#+0W ,#P0W5-E)0'07
Câu 338:d66>!
>
A

J[I'+1Z Z*A5I 7IK26 !6!#Ea:
689??SE

$A
U
Iw5?K7IK20'PZ Z$A

5\z6 ?3hI 7#A>!
>
A


&#*

A
"
# )#CA# *#C
"
A
U
# ,#*A#5-E)0'07
- Fe, Cu tác dụng HNO
3
, H

2
SO
4 đặc
Câu 339: *v2C*\=H!??SEBA
"

Y#$ !\=IK2??S
<=63 !6!?K#*3I
&#C5BA
"
7
"
)#C5BA
"
7

*#EBA
"
#,#*5BA
"
7

# !/$-E*- !)00+
Câu 340:
*v2R6C??S>!E$AU

IwI: !F\=
>\
IK2??S46JC @#*3??S4
&#$A

U
C

5$A
U
7
"
# )#$A
U
C

5$A
U
7
"

C$AU
#
*#$AUC$AU# ,#$AU#
!/$*-00+
Câu 341:
*P+6C??S=0"6E$AUIw5!\!:$A\z6
 ?37#
$ !\=>\IK2
&#0'6
C$AU
# )#006C

5$A
U

7
"

00(6
C$AU
#
*#006
C

5$A
U
7
"

006C?K#,#00"6
C

5$A
U
7
"


00P6C$AU#
!/$-E*- !)00+
Câu 342: /qZ??SEBA
"
'5Y7Z3J?`Iq6v2R60'6C
0'6*58!:\=3 ?3BA7
&#0(Z# )#'0Z# *#0PZ# ,#'Z# !/$-E*- !)00(

Câu 343:
Ea'6v2C*5<%6'L'789>!EBA"IK2eZ5I 7v2 Z
5R6BABA7??S45<=!6!>!?K7#/< !

I!H!E

89'1#}!FSe
&#U# )#UU(# *#P0# ,#""P# !/$-E*- !&00+
Câu 344: *P+6CU006??SEBA
"

'I: !\=>\IK2 ZBA
5\z6 ?37??S#,?Sq!I66*#}!FS6
&#'1# )#"0# *#0PU# ,#"(U# !/$-E*- !&001
Câu 345: *P'6v2R6*C
"
A
U

F?•H!??SEBA
"

YI 3I.#
$ !F\=>\IK2""PZ ZBA5\z6 ?3hI 7??S4a
!U6 !6!#*@??S4IK2666! #}!FS6
&#''# )#'"+'# *#1+# ,#'0(1# !/$-E*- !)001
- Hợp chất của Fe
Câu 346: /F!uwmuw6K2m3
&#6!I~# )#>!I!# *#6!G#,#6!# !/$-E*- !&00(
Câu 347: d!  v 2 F 3 C5BA

"
7

 C5AE7
"

 C*A
"
 @ Z I: ! K2 @I!
IK263m
&#C"AU# )#CA# *#C# ,# CA"#
 !  /$-E*-  !  &
00+
Câu 348: Ea  C
"
A
U

 ? ?S E

$A
U
Y 5?K7 IK2? ?S
'
#* K2
?K8C
??S'

5I!. !% @ @ Z7I: !\=>\


IK2??S

=3

&#C5$AU7"E$AU# )#C$AU#
*#C5$AU7"# ,#C$AUE$AU#
!/$*-00(
Câu 349: *?YF3LCAC5AE7


C$AU
C
"
A
U
C

5$A
U
7
"
C

A
"
#$3?Y8S>! !F?•
H!??SEBA
"
Iw
&#"# )## *#U ,#P# !/$*-00(

Câu 350: /F3LC*

C*
"
C5BA
"
7

C5BA
"
7
"
C$A
U
C

5$A
U
7
"
#$3\Z>!FZ

&## )#"# *## ,#U# !/$*-001
Câu 351: *[3LCCAC5AE7

C5AE7
"
C
"
A

U
C

A
"
C5BA
"
7

C5BA
"
7
"

C$AU
C

5$A
U
7
"
C*A
"

JK2
\=H!EBA
"
Iw#$\=!\=>!Fc 
Trang 24/59 – Các dạng bài tập Hóa học ôn thi ĐH, CĐ từ 2007 đến 2011
&#(# )## *#+# ,#P# !/$-E*- !&00+

Câu 352:
EP6C89??SE$AU

Y5?K7IK2??S#,?S

\=[I
H!e6??SdAU0#}!FSe
&#(0# )#U0# *#0# ,#P0# !/$-E*- !&00+
Câu 353: -q     " 6 v 2 R6 CA C
"
A
U
C

A
"

5 I  6 CA
896
C

A
"
7J?`[IeZ??SE*'#}!FSe
&#0'P# )#0'(# *#0"# ,#00(# !/$-E*- !&00(
Câu 354: *1'6v2R6CAC
"
A
U
C


A
"

F?•H!??SE*5?K7#$ !F\=>\
IK2??S4N@4IK2+P6C*

66
C*
"
#}!FS6
&#(+# )#1+# *#P0# ,#+(0# !/$-E*- !)00(
Câu 355: *66v2R6CAC

A
"
C
"
A
U
6K2[I??SE*IK2?
?S4<%6C
b
C
"b
'L#*!4!J89#*@J6IK26
'
66!
#$• Z5?K7J!@??S\=IK26


66! #)!:6

ƒ6
'
M0+'#
/qZ??SE*IY?`
&#U06# )#(06# *#"06# ,#'P06# !/$*-001
Câu 356: * ''"P 6 v 2 R6 C CA C

A
"
C
"
A
U

\ = : H! ? ?SEBA
"
Y5?K7
IK2'"UUZ ZBA5\z6 ?3hI 7??S#*@??SIK2666! #
}!FS6
&#"0# )#"U"P# *#U101# ,#"(+# !/$-E*- !&00(
Câu 357: Ea0((66>!m89??SE

$A
U

IwIK2??S"U(
Z Z$A



5\z6 ?3hI 7#*@??SIK2666!y #}!FS6
&## )#U0# *#(0# ,#U(U# !/$-E*- !)001
Câu 358: 
*00'6623mF?•:H!E$AU

Iw5?K7F0''Z

5I 7 Z$A
\z6 ?37#*@=23mI
&#C$# )#C$# *#CA,#C*A"#
!/$-E*- !)00+
Câu 359: d6>!mh!%IJ[IeZ Z*A5hI 7\=IK20(U
6C006 Z*A

#*@=!FSeJK2
&#C
"
A
U

0U# )#C
"
A
U

0UU(# *#CA0U# ,#C

A
"


0UU(#
!/$*-001
Câu 360: *UU(Z Z*A5hI 7[[I!u=IQ(66>!mI: !\=>\
#dZIK2\=< !H!!I890#*@=>!mJV6qZ Z
*A


v2 Z\=
&#C"AUN+W# )#CA"N+W# *#CA"NPW# ,#CAN+W#
!/$*-00+
Câu 361: *66??S=0'6C*
"
#$ !\=>\IK2""P6
3m#}!FS6
&#'P# )#0U# *#U"# ,#((# !/$*-001
Câu 362: *'006??SC*


'F?•H!006??S&BA
"

IK266 :#
}!FS6
&#"UUU# )#U+U# *#"0'(# ,#'1P# !/$*-001
Câu 363: EUU6v2R6C*


B*5<%6K='L7
6K2

KH5?K7IK2??S#*??S&BA
"

5?K7??S
 !\=>\!
663m#}!FS6
&#+U# )#(+# *#'0(# ,#P(# !/$-E*- !)001
Câu 364: B6v2mR66C*A
"

86
C$
8T Z= @ Z5?K7#$ ! F
\=>\IK8T.!%I8IJIK23m?3C

A
"

v2 Z#)!:F
3 Z8TKH\=896!!%!n858!:F\=Kfh6=
>!FbU7
&#MU8# )#M8# *#M8#,#M08#
!/$-E*- !)00(
Câu 365: /P 68 mH! U68 KfR!  5I!. !% @  @ Z7
IK2 v 2 m # *  F ?• H! K2 ?K ? ?S E* !\!  v2 Za!6J
@}#-qIF}J[IeZ ZA


5hI 7#}!FSe
&#(0# )#""P# *#"0(# ,#UU(# !/$*-00(

Câu 366:Ev2R606C06C

A
"
??S>!E

$A
U
Y5?K7IK2
UZ Z5I 7??S4#*K2?K??SBAE??S4 !\=>\
IK266 :#}!FS~36L
&#U0 )#1' *#P0( ,#+U5*-0''7
Câu 367:*v2R6*&C&F?•H!>!?K !IIK23m4#*4??S
E*?K 3 ‘I3??SIK2F?•H!??SBAEY?K#D;3 :
I6 @ ZI: !K2 @I!IK23mX#)!:F\=>\aa#/J
XR6L
Trang 25/59 – Các dạng bài tập Hóa học ôn thi ĐH, CĐ từ 2007 đến 2011

×